Tài liệu học tiếng anh -...

122
TÀI LIỆU HỌC TIẾNG ANH ĐỌC HIỂU 1 (PHẦN BỔ SUNG) DÀNH CHO SINH VIÊN (NĂM HỌC 2010 – 2011) Chapter 1: READING FOR DETAILS Introducing Details PREREADING QUESTIONS Answer the following questions 1. What do you know about hairstyles in ancient times? 2. What kinds of things to do you think people did to their hair in ancient times 3. Do you think that hairstyles and colors were important for both men and women? People have been concerned with their hair since ancient times. In 1500 B.C., the Assyrians, inhabiting the area know today as Northern Iraq, were the world's first true hairstylists. Their skills at cutting, curling, layering and dyeing hair were known throughout the Middle East. In fact, they were obsessed with their hair, which was oiled, perfumed, and tinted. A fashionable courtier wore his hair cut in neat geometric layers. Kings, soldiers, and noblewomen had their hair curled with a fire-heated

Transcript of Tài liệu học tiếng anh -...

Page 1: Tài liệu học tiếng anh - saomaidata.orgsaomaidata.org/library/306.TaiLieuHocTiengAnh.docx  · Web viewTÀI LIỆU HỌC TIẾNG ANH. ĐỌC ... layering and dyeing hair were

TÀI LIỆU HỌC TIẾNG ANHĐỌC HIỂU 1 (PHẦN BỔ SUNG)

DÀNH CHO SINH VIÊN

(NĂM HỌC 2010 – 2011)

Chapter 1: READING FOR DETAILSIntroducing Details

PREREADING QUESTIONS

Answer the following questions

1. What do you know about hairstyles in ancient times?

2. What kinds of things to do you think people did to their hair in ancient times

3. Do you think that hairstyles and colors were important for both men and

women?

People have been concerned with their hair since ancient times. In 1500 B.C.,

the Assyrians, inhabiting the area know today as Northern Iraq, were the

world's first true hairstylists. Their skills at cutting, curling, layering and dyeing

hair were known throughout the Middle East. In fact, they were obsessed with

their hair, which was oiled, perfumed, and tinted. A fashionable courtier wore

his hair cut in neat geometric layers. Kings, soldiers, and noblewomen had

their hair curled with a fire-heated iron bar, probably the world's first curling

iron. So important was hair styling in Assyria that law dictated certain types of

hairstyles according to a person's position and employment. Facial hair was

also important. Men grew beards down to their chests and had them clipped in

layers. High-ranking women in both Egypt and Assryia wore fake beards

during official court business to show their equal authority with men.

Like the Asyrians, the early Greeks liked long, scented, curly hair. Fair

hair was favored over dark, so those who were not "natural blonds" Ughtened

Page 2: Tài liệu học tiếng anh - saomaidata.orgsaomaidata.org/library/306.TaiLieuHocTiengAnh.docx  · Web viewTÀI LIỆU HỌC TIẾNG ANH. ĐỌC ... layering and dyeing hair were

or reddened their hair with soaps and bleaches. The Romans, on the other

hand, favored dark hair for men for high societies, or political rank. Early

Saxon men were neither blonds nor brunets but dyed their hair and beards

blue, red, green, and orange.

Over the centuries, societies have combed, curled, waved, pow-dered,

dyed, cut, coiffed, and sculpted their hair, or someone else's during times of

wig crazes. Churches and lawmakers have sometimes tried to put a stop to

the human obsession with hair, but with little success. It seems hairstyling is

here to stay, and the future will likely prove no exception.

EXERCISE 1Answer the following questions.

SKIMMING

Read the passage quickly once again.

1. What is the passage about?

SCANNING

Look over the article again to find the answers to questions 2-15.

Complete the following sentences with details from the passage.

2. The hairstyling skills of the Assyrians were known all over _____

3. An Assyrian courtier had his hair ___

4. The Assyrians had laws for certain types of hairstyles according to people's

___ and ___

5. Dunring official court business women in Egypt wore ___

6. ___ preferred fair hair.

7. ___ preferred dark hair for men of high rank.

Locate the following details in the passage. Give the line numbers.

8. In which lines does the author explain how people curled their hair?

Page 3: Tài liệu học tiếng anh - saomaidata.orgsaomaidata.org/library/306.TaiLieuHocTiengAnh.docx  · Web viewTÀI LIỆU HỌC TIẾNG ANH. ĐỌC ... layering and dyeing hair were

9. In which lines does the author first mention changing the color of hair?

10. At what point in the passage does the author discuss the wearing of wigs?

Underline the detail that is NOT mentioned in the passage in each of the

sentences below.

11. The kings, soldiers, and women of Assyria curled their hair.

12. The Assyrians and the Grecks liked long, perfumed, blond, curly hair.

13. Beards were important for the Assyrians, Egyptians, and Greeks.

Detail Questions

Detail questions ask you about specific information in the passage. Detail

questions usually begin with the words

ACCORDING TO THE PASSAGE...

To answer detail questions, focus on the key word or words used in the

question Then you must scan the passage. When you scan a passage, you

move your eyes quickly over the passage until you find the key words that you

are looking for: a name, a date, a number. It is not necessary to read the

whole passage again-just locate the key words. Once you find the key words,

you can read the sentences that follow or come before to make sure you have

found the right information.

The correct answer to a detail question will not usually use the exact

words as found in the passage but synonyms or a restatement of what is

stated in the passage. For example, if the passage states that "Eugene O'Neill

was a well known dramatist," the answer to a question about the kind of work

he was known for might state that “lhis plays won him fame”

Detail questions usually appear in the order of the information

presented in the passage. This means that the answer to the first detail

question will come near the beginning of the passage and the information for

the second question will come after that.

Sample Reading Passage

Page 4: Tài liệu học tiếng anh - saomaidata.orgsaomaidata.org/library/306.TaiLieuHocTiengAnh.docx  · Web viewTÀI LIỆU HỌC TIẾNG ANH. ĐỌC ... layering and dyeing hair were

Although "lie detectors" are being used by governments, police

departments, and businesses that all want guaranteed ways of detecting the

truth the results are not always accurate. Lie detectors are properly alled

emotion detectors, for their aim is to measure bodily changes that contradict

what a person says. The polygraph machine records changes in heart rate,

breathing, blood pressure, and the electrical activity of the skin (galvanic skin

response, or GSR). In the first part of the polygraph test, you are electronically

connected to the machine and asked a few neutral questions ("What is your

name?" "Where do you live?”). Your physical reactions serve as the standard

(beseline) for evaluating what comes next. Then you are asked a few entical

questions among the neutral ones (“When did you rob the bank?”). The

assumption is that if you are guilty, your body will reveal the truth, even if you

try to deny it. Your heart rate, respiration, and GSR will change abruptly as

you respond to the incriminating questions.

That is the theory; but psychologists have found that lie detectors are

simly not reliable. Since most physical changes are the same across all

emotions, machines cannot tell whether you are feeling guilty, angry, nervous,

thrilled, or revved up from an exciting day. Innocent people may be tense and

nervous about the whole procedure. They may react physiologically to a

certain word ("bank") not because they robbed it, but because they recently

bounced a check. In either case the machine will record a “lie”. The reverse

mistake is also common. Some practiced liars can lie without flinehing, and

others learn to beat the machine by tensing muscles or thinking about an

exciting experience during neutral questions.

QUESTION

1. According to the passage, polygraph tests

(A) record a person's physical reactions

(B) measure a person's thoughts

(C) always reveal the truth about a person

Page 5: Tài liệu học tiếng anh - saomaidata.orgsaomaidata.org/library/306.TaiLieuHocTiengAnh.docx  · Web viewTÀI LIỆU HỌC TIẾNG ANH. ĐỌC ... layering and dyeing hair were

(D) make guilty pcople angry

ANSWER

Answer (A) is correct because it is a rewording of "bodily changes". Answer

(B) is incorrect because the polygraph measures physical changes; thoughts

are not physical changes. Answer (C) is also incorrect since the passage

states that lie detectors are "simply not reliable." Answer (D) is incorrect since

the polygraph does not make guilty people nervous; it makes innocent pcople

nevous.

QUESTION

1) According to the passage, what kind of questions are asked on the first part

of the polygraph test?

(A) Critical

(B) Unimportant

(C) Incriminating

(D) Emotional

ANSWER

Answer (A) is not correct because critical questions are asked on the second

part of the test. Answer (C) is also not conrrect since incriminating questions

are not asked on the first part of the test. Answer (D) is also incorrect since

"What is your name? and "Where do you live?" are not emotional questions.

The best answer is (B), which is another word for "neutral."

DETAIL QUESTIONS ABOUT WHAT IS NOT IN THE PASSAGE

This type of detail question asks about what is not in the passage or

what is not true according to the passage. These questions have the word

NOT or EXCEPT in capital letters. The following are examples of such

questions:

Which of the following is NOT mentioned in the passage?

Page 6: Tài liệu học tiếng anh - saomaidata.orgsaomaidata.org/library/306.TaiLieuHocTiengAnh.docx  · Web viewTÀI LIỆU HỌC TIẾNG ANH. ĐỌC ... layering and dyeing hair were

According to the passage, all of the following are true EXCEPT...

In this type of question, three of the answers are true and one of the

answers is not mentioned in the passage or is not true. Scan the passage to

find the answers that are true or stated in the passage. The answer that is not

mentioned in the passage or is not true is the correct one.

Read the sample reading passage again.

QUESTION

1. Which of the following is NOT mentioned as something that is measured by

a polygraph machine?

(A) Blood pressure

(B) Heart rate

(C) Breathing

(D) Eye movement

Rememher, three of the answers are mentioned and one is not mentioned.

ANSWER

(A) is mentioned because the passage states that the polygraph records

changes in blood pressure. Because (A) is mentioned, it is not the correct

answer. (B) is also mentioned because the passage states that the polygraph

machine records changes in heart rate. Since (B) is mentioned, it is not

correct. Answer (C) is also mentioned, because the passage states that the

polygraph machine records breathing. Since (C) is mentioned it is not correct.

The best answer to the question is therefore (D) because it is not mentioned.

Straregies for Answering Detail Questions

- The answers to detail questions will follow the order of information presented

in the passage.

- The correct answers to detail questions are often a restatement of what is

stated in the passage.

Page 7: Tài liệu học tiếng anh - saomaidata.orgsaomaidata.org/library/306.TaiLieuHocTiengAnh.docx  · Web viewTÀI LIỆU HỌC TIẾNG ANH. ĐỌC ... layering and dyeing hair were

- If the question has the word NOT or EXCEPT, choose the answer that is not

true or not mentioned in the passage. Answers that are true or mentioned in

the passage are not correct.

Exercises on Details

EXERCISE 2Read the passage and answer the detait quetions that follow each one.

QUESTIONS 1-6

Hermit crabs occupy the empty shells of dead sea snails for protection

while still retaining their mobility. They are capable of discriminating among a

selection of shells of vanous sizes and species, and they choose the one that

fits the body most closely. Hermit crabs change shells as their grow, although

in some marine environments a large enough variety of shells may not be

available and the hermit crab may be forced to occupy a smaller-than-ideal

"house". When a shell becomes too small for the hermit crab to occupy, it will

sometimes become aggressive and fight other hermit crabs to gain a larger

shell.

Hermit crabs may encounter empty shells in the course of their daily

activity, but the vacant shell is usually sponed by sight. The hermit crab's

visual response increases with the size of an object and its contrast against

the background. The hermit crab then scizes the shell with its walking legs

and climbs on it, monitoring its size. If the size is right, the crab investigates its

shape and texture by rolling it over between its walking legs and running its

claws over the surface. Once the shell's opening has been located, the crab

uses its claws to remove any foreign material before preparing to enter. The

crab rises above the opening, flexes its abdomen, and enters the shell

backward. The shell intenor is monitored by the abdomen as the crab

repcatedly enters and withdraws. When conrpletely satisfied with its new

Page 8: Tài liệu học tiếng anh - saomaidata.orgsaomaidata.org/library/306.TaiLieuHocTiengAnh.docx  · Web viewTÀI LIỆU HỌC TIẾNG ANH. ĐỌC ... layering and dyeing hair were

mobile home, the hermit crab will emerge one last time, turn the shell over and

make a final entrance.

1. According to the passage, hermit crabs occupy vacant shells for

(A) mobility

(B) flexibility

(C) protection

(D) discrimination

2. According to the passage, a hermit crab changes shells when it

(A) outgrows the one it has

(B) hunts fur food

(C) becomes aggressive

(D) locates any vacant shell

3. According to the passage, the way in which hermit crabs locate empty

shells is through which of the following senses

(A) Hearing

(B) Touch

(C) Taste

(D) Sight

4. A crab investigates a vacant shell for all of the following EXCEPT

(A) size

(B) type

(C) shape

(D) texture

5. According to the passage, a hermit crab enters a new shell

(A) head first

Page 9: Tài liệu học tiếng anh - saomaidata.orgsaomaidata.org/library/306.TaiLieuHocTiengAnh.docx  · Web viewTÀI LIỆU HỌC TIẾNG ANH. ĐỌC ... layering and dyeing hair were

(B) claws first

(C) backward

(D) with its walking legs

6. According to the passage, a hermit crab senles into its new "mobile home"

(A) after entering and leaving several times

(B) without inspecting the intenor first

(C) immediately after locating the shell opening

(D) after fighting other hermit crabs for al larger shell

QUESTIONS 7-13

The first black literature in America was not written but was preserved in

an oral tradition, in a rich body of folklore, songs and stones, many from

African origins. There are humorous tales, Biblical stones, animal stories, and

stones of natural phenomena, of good and bad people, and of the wise and

foolish. Many reflect how African Americans viewed themselves and their

lives. The lyrics of blues, spirituals and work songs speak of suffering and

hope, joy and pain, loved ones, and religious faith, and are an integral part of

the early litenture of black people in America.

The earliest existing written black literature was Lucy Teny's poem

"Bars Fight." written in 1746. Other eighteenth-century black poets include

Jupiter Hammon and Geoge Moses Horton. The first African American to

publish a book in American was Phillis Wheatley. Slack poetry also flourished

in the nineteenth-century, during which the wntings of almost forty poets were

printed, the most notable of whom was Paul Laurence Dunbar, the first black

American to achieve national acclaim for his work. Dunbar published eight

volumes of poetry and eight novels and corrections of stones.

More than three dozen novels were written by blacks between 1855 and

1899, but autobiography dominated African-American literature in the

nineteenth-century, as it had in the eighteenth. In the twentieth century,

Page 10: Tài liệu học tiếng anh - saomaidata.orgsaomaidata.org/library/306.TaiLieuHocTiengAnh.docx  · Web viewTÀI LIỆU HỌC TIẾNG ANH. ĐỌC ... layering and dyeing hair were

however, fiction has presided, with Charles W. Chestnutt, America's first black

man of letters, successfully bridging the two centuries. He began publishing

short fiction in the mid-1880s, wrote two books that appeared in 1899, and

had three books published between 1900 and 1905. He was a pioneer of the "

new literature" of the early 1900s, which aimed to persuade readers of the

worth and equality of African Americans.

7. Which of the following is NOT mentioned in the passage as part of the oral

tradition of African Americans?

(A) Humorous tales

(B) Tales of adventure

(C) Bibhcal stories

(D) Animal stones

8. According to the passage, the lyries of blues and spirituals are often

concerned with

(A) the pain and ioy in life

(B) loved ones and animals

(C) religion and nature

(D) wise and foolish people

9. According to the passage, an important part of early African-amerian

literature was

(A) novels

(B) short fiction stories

(C) biographies

(D) songs

10. According to the passage, when did the first written African-amrincan

literature appear?

Page 11: Tài liệu học tiếng anh - saomaidata.orgsaomaidata.org/library/306.TaiLieuHocTiengAnh.docx  · Web viewTÀI LIỆU HỌC TIẾNG ANH. ĐỌC ... layering and dyeing hair were

(A) In the l600s

(B) In the 1700s

(C) In the 1800s

(D) In the 1900s

11. According to the passage, who was the first African American to receive

national recognition for his writing?

(A) Paul Dunbar

(B) George Horton

(C) Lucy Tenv

(D) Phiuis Wheatley

12. According to the passage, what form dominated African-American

literature in the nineteenth century?

(A) Poetry

(B) Novels

(C) Autobiography

(D) Fiction

13. According to the passage, Charles W. Chestnutt was one the first writers

to

(A) write about the suffering of African Americans

(B) publish short fiction in the early 1990s

(C) write persuasively about the worth of African Americans

(D) dominate the African-American literary tradition

QUESTION 14-20

A Snowflake onginates from countlees water moleules that iunitially

come together in small groups as a result of a weak attractive force between

oxygen and hydrogen atoms. The same forces subsequendy organize the

Page 12: Tài liệu học tiếng anh - saomaidata.orgsaomaidata.org/library/306.TaiLieuHocTiengAnh.docx  · Web viewTÀI LIỆU HỌC TIẾNG ANH. ĐỌC ... layering and dyeing hair were

groups into a frozen molecular crystal, a perfectly organized lattice of

molecules. Finally, sevveral molecular crystals join to form a snowflake.

Scientists have realized for some time that the forces that assemble

molecules into natural crystals can be utilized to produce a variety of

imnportant materials. They have detemlined the structure of more than 90,000

different molecular crystals, the most common example of which are aspirin

and mothballs.

In recent years, researchers have studied how molecules organize

themselves to fom crystals in the hope of better understanding what types of

mokcules and what conditions, will produce molecular crystals with unusual

and useful properties. Scientists are aware that the material properties of a

crystal depend in large part on the organization of the molecules in the cyrstal,

yet they know linle about the factors controuing the assembly of such crystals.

Synthesizing a molecular crystal is similar to designing a building.

Before construction can begin, the architect must specify the shapes and

sizes of the girders and the number and placement of the rivets. Similarly, to

produce new molecular crystals, chemists must choose molecules of the

appropriate sizes and shapes and select the molecular forces that will hold the

crystals together. A chemist can normally find many molecules of vanous

shapes and sizes, but the challenge is to find ones that assemble in a

predictable manner.

14. According to the passage, a snowflake is formed by

(A) the attractive force between oxygen and hydrogen

(B) molecular crystals with new and useful propen'es

(C) the synthesizing of molecular crystals

(D) the joining of several molecular crystals

15. According to the passage, water molecules join together as a result of

(A) an attraction between oxygen and hydrogen atoms

Page 13: Tài liệu học tiếng anh - saomaidata.orgsaomaidata.org/library/306.TaiLieuHocTiengAnh.docx  · Web viewTÀI LIỆU HỌC TIẾNG ANH. ĐỌC ... layering and dyeing hair were

(B) the organization of the molecules in a crystal

(C) a strong force that assembles crystal atoms

(D) the unusual and useful properties of molecular crystals

16. By making use of forces that assemble molecules into natural crystals,

scientists can

(A) find molecules of various shapes and sizes

(B) detemine the structure of different molecular crystals

(C) organize molecules into a perfect lattice

(D) create new and useful materials

17. According to the passage, what reason do researchers have for studying

how molccules organize themselves to form crystals?

(A) To assemble molecules into natural crystals.

(B) To leam how to synthesize molecular crystals

(C) To make aspirin and mothballs

(D). To change the material properties of a crystal

18. According to the passage, what do scientists still need to learn about the

organization of molecules?

(A) What detemines the material property of a crystal

(B) The molecular forces that hold molecules together

(C) The conditions that produce molecular crystals.

(D) The factors controlling the way crystals are assembled...

19. To produce new molecular crystals, chemists must choose all of the

following EXCEPT

(A) molecules of the right size

(B) molccules of the appropriate shape

Page 14: Tài liệu học tiếng anh - saomaidata.orgsaomaidata.org/library/306.TaiLieuHocTiengAnh.docx  · Web viewTÀI LIỆU HỌC TIẾNG ANH. ĐỌC ... layering and dyeing hair were

(C) the right molecular organization

(D) the proper molecular forces

2O. According to the passage, the task of synthesizing a molecular crystal can

be compared to

(A) designing a building

(B) building a house

(C) making materials

(D) constructing a lattice

QUESTIONS 21-28

Education was of primary importance to the English colonists and was

conducted at home as well as in established schools. Regardless of

geographic location or finances, most Americans learned to read and compute

numbers. For many, the Bible and other religious tracts were their only books;

however, the excellent language contained in such works usually made them

good primers. Many families owned one or more of Shakespcare's works, a

copy of John Bunyan's dassic A Pilgrim’s Progress, and sometimes collections

of English literary essays, poems, or historical speeches.

In 1647 the Massachusetts School Law required every town of at least

50 households to maintain a grammar school. The law was the first to

mandate public education in America. In the middle colonies at the time,

schools were oftẹn dependent on rehgious societies, such as the Quakers and

other private organizations. In the South, famihes employed private tutors or

relied on the city to conduct education. At the outset, most elementary schools

were for boys, but schools for girls were established in the eighteenth century

in most cities and large towns. In spite of the informal atmosphere of most

American schools, the literacy rate in the colonies of mid-eighteenth-century

America was equal to or higher than in most European countries.

Page 15: Tài liệu học tiếng anh - saomaidata.orgsaomaidata.org/library/306.TaiLieuHocTiengAnh.docx  · Web viewTÀI LIỆU HỌC TIẾNG ANH. ĐỌC ... layering and dyeing hair were

Before the American Revolution, nine colleges had been founded,

including Harvard, William and Mary, Yale, the Colleges of New Jersey (now

Princeton), Brown, Rutgers, Dartmouth, and Kings College (later Columbia

University). By 1720 the natural sciences and modem languages were being

taught, as well as courses in practical subjects such as mechanics and

agriculture. At the end of the eighteenth century, medical schools were

established at the College of Philadelphia and at King's Colleges.

21. Which of the following words best describes the English colonists attitude

toward education?

(A) Indifferent

(B) Distrustful

(C) Enthusiastic

(D) Casual

22. According to the passage, most Americans learned how to

(A) write

(B) read

(C) farm

(D) speak a foreign language

23. According to the passage, all of the following sometimes substituted for

school books EXCEPT

(A) historical speeches

(B) works of Shakespeare

(C) literary essays

(D) biographies

24. According to the passage, the Massachusetts School Law applied to

every town with how many households?

Page 16: Tài liệu học tiếng anh - saomaidata.orgsaomaidata.org/library/306.TaiLieuHocTiengAnh.docx  · Web viewTÀI LIỆU HỌC TIẾNG ANH. ĐỌC ... layering and dyeing hair were

(A) Less than fifty

(B) Exactly fifty

(C) Fifty or more

(D) Fifteen

25. According to the passage, the middle colonies often depended upon which

group to provide education?

(A) Private organizations

(B) Colleges.

(C) Established primary schools

(D) Businesses

26. According to the passage, who often conducted education in the South?

(A) Public school teachers

(B) Doctors

(C) Clergy

(D) Politicians

27. How well educated were Americans in comparison to most European

countries?

(A) Much worse

(B) The same or better

(C) Far better

(D) Less or equal

28. According to the passage, all the following subjects are mentioned as

being taught in colleges in the 1700s EXCEPT

(A) languages

(B) science

Page 17: Tài liệu học tiếng anh - saomaidata.orgsaomaidata.org/library/306.TaiLieuHocTiengAnh.docx  · Web viewTÀI LIỆU HỌC TIẾNG ANH. ĐỌC ... layering and dyeing hair were

(C) medicine

(D) economies

QUESTIONS 29-36

The study of control processes in electronic, mechanical, and blological

systems is known as cybernetics. The word was coined in 1948 by the

American mathematician Norbert Wiener from the Greek word meaning pilot

or steersman. Cybernetics is concerned with the analysis of the flow of

information in both living organisms and machines, but it is particularly

concerned with systems that are capable of regulating their own operations

without human control

Automatic regulation is accomplished by using information about the

state of the end production that is fed back to the regulating device, causing it

to modify or conrrect production procedures if necessary. The concept of

feedback is at the very heart of cybernetics and is what makes a system

automatic and self-regulating. A simple example of a self-regulating machine

is a thermostat, which reacts to continual feedback about the outside

temperature and responds accordingly to achieve the temperature that has

been programmed into it.

The applications of cybernetics are wide reaching, appearing in science,

engineering, technology, sociology, economies, education, and medicine.

Computers can keep a patient alive during a surgical opention, making

instantaneous modifications based on a constant flow of information. In

eduation, teaching machines use cybernetic principles to instruct students on

an individual basis. In the home, automation is present in such everyday

products as refrigentors, coffeemakers, and dishwashers. In industry,

automation is increasing its applications, although it is currenưy applied

primarily to the large-scale production of single units. In industries in which a

break in the flow of production can ruin the product, automatic controls are

invaluable. Chemical and petroleum plants are now almost completely

automatic, as are industries involved in the production of chemicals and

Page 18: Tài liệu học tiếng anh - saomaidata.orgsaomaidata.org/library/306.TaiLieuHocTiengAnh.docx  · Web viewTÀI LIỆU HỌC TIẾNG ANH. ĐỌC ... layering and dyeing hair were

atomic energy. Automation has become the answer when human safety is the

number one priority.

29. Cybernetics is the study of control processes in all of the systems

EXCEPT

(A) ecological

(B) biological

(C) mechanical

(D) electronic

30. According to the passage, the word "cybernetics" was coined from the

Greek word meaning

(A) information

(B) automatic

(C) pilot

(D) regulator

31. According to the passage, cybernetics is primarily concerned with systems

that

(A) are controued by humans

(B) analyze flaws of information

(C) are self-regulating

(D) have wide-reaching applications

32. According to the passage, how is automatic regulation accomplished?

(A) By modifying and correcting production procedures

(B) By feeding information to the regulatory device

(C) By analyzing the flow of information to the organism

(D) By making modifications in cybernetic principles

Page 19: Tài liệu học tiếng anh - saomaidata.orgsaomaidata.org/library/306.TaiLieuHocTiengAnh.docx  · Web viewTÀI LIỆU HỌC TIẾNG ANH. ĐỌC ... layering and dyeing hair were

33. According to the passage, what makes a system automatic and self-

regulating?

(A) Information

(B) Production procedures

(C) Human control

(D) Fcedback

34. Which of the following is NOT mentioned as an area in which cybernetics

has applications?

(A) Technology

(B) Engineering

(C) Philosophy

(D) Education

35. According to the passage, automation in industry is primarily used in

producing

(A) large quantities of a single unit

(B) everyday household products

(C) small amounts of many different products

(D) high-tech surgical instruments

36. According to the passage, automation is extremely important when the top

priority is

(A) efficiency

(B) speed

(C) convenience

(D) safety

QUESTIONS 37-44

Page 20: Tài liệu học tiếng anh - saomaidata.orgsaomaidata.org/library/306.TaiLieuHocTiengAnh.docx  · Web viewTÀI LIỆU HỌC TIẾNG ANH. ĐỌC ... layering and dyeing hair were

Cattle ranchers throughout the American West owe much of their

traditional culture to the Spaniards, who first introduced cattle to the New

World and first developed cattle ranching in the Westem Hemisphere. The

vaquero, or Mexican cowboy, was born of the necessity to look after the cattle

that grazed open ranges. He was not a romantic figure but a poor laborer on

horseback, who wore what clothes he had on his back and eventually found

certain types of dress more appropriate than others, a blend of Spanish dress

and that worn by the natives.

Working in the hot sun brought the adoption of Spanish sombreros and

bandannas. Because it was waterproof and wind resistant, leather was

eventually the chosen material for lackcts and leggings, or botas, the

predecessor to chaps. A large pair of iron spurs were the badge of the

vaquero, and a lazo (lasso) - a rope with slipknot - was the vaquero's primary

working tool, especially on the trail drives that became commonplace by the

the sixteenth century. Saddle makers added a large saddle horn to the

Spanish saddle to accommodate the lasso during the roping technique, in

which the vaquero tossed the rope around the cow and then quickly tied and

wrapped the end of the rope around the horn. Later, American cowboys north

of the Rio Grande learned this technique.

As the size of cattle herds grew and rustlers became a problem, the

Spanish cattlemen asked the authorities to put a stop to them. The Spanish

crown responded with the establishment of the Mesta to enact ordinances to

beneat and increase the herds and to remedy and punish crimes. The Mesta

selved the special interests of cattle raisers and preceded the American

West's cattlemen's associations. Moreover the Mesta's ordinances were

similar to modem American laws relating to ranching, and today's laws, in fact,

are essentially variations and adaptations of the regulations first established in

the New World more than four centuries ago.

37. According to the passage, which of the following best describes the

vaquero?

Page 21: Tài liệu học tiếng anh - saomaidata.orgsaomaidata.org/library/306.TaiLieuHocTiengAnh.docx  · Web viewTÀI LIỆU HỌC TIẾNG ANH. ĐỌC ... layering and dyeing hair were

(A) A romantic figure

(B) A wealthy cattle rancher

(C) A poor working man

(D) A Spanish explorer

38. According to the passage, who introduced cattle to the Westem

Hemisphere?

(A) The American cowboys

(B) The Spaniards

(C) The Mexicans

(D) The native Americans

39. According to the passage, what kind of clothing was worn by the Mexican

cowboy?

(A) A combination of formal and infomlal dress

(B) A variation on the style worn by American cattlemen

(C) The same type of garments the cowboys wore in their villages

(D) A blend of native and Spanish dress

40. According to the passage, what element of nature inspired the vaqueros to

wear hats and bandannas?

(A) Sun

(B) Wind

(C) Rain

(D) Cold

41. According to the passage, leather was chosen as the material for a

cowboy's jacket and leggings because

(A) it was warm

Page 22: Tài liệu học tiếng anh - saomaidata.orgsaomaidata.org/library/306.TaiLieuHocTiengAnh.docx  · Web viewTÀI LIỆU HỌC TIẾNG ANH. ĐỌC ... layering and dyeing hair were

(B) it was waterproof

(C) it made good padding for horseback riding

(D) it was good protection from the sun

42. According to the passage, which of the following is NOT mentioned as

being among a vaquero's prossessions during trail drives?

(A) A pair of spurs

(B) A lass.

(C) A pair of botas

(D) A pair of leather gloves

43. According to the passage, why did saddle makers eventually put a horn on

the Spanish saddle?

(A) To make a place on which to tie the lasso

(B) So the cowboy would have something to hold onto

(C) To add something to hang things on

(D) To make it easier to get on and off the horse

44. According to the passage, the ordinances enacted by the Mesta

(A) were meant to protect the working cowboy

(B) protected the rights of the natives

(C) were to serve the interests of the cattle ranchers

(D) were eliaminated when American ranching laws were passed

QUESTIONS 45-49

May 18, 1980, dawned clear and cool in the Cascades, but it would

pass into history as a momentous day when the cataclysmic eruption of Mt.

St. Helens turned a vast area of the pristine Washington countryside into a

cauldron of devastation. Mt. St. Helens was one of the most beautiful

Page 23: Tài liệu học tiếng anh - saomaidata.orgsaomaidata.org/library/306.TaiLieuHocTiengAnh.docx  · Web viewTÀI LIỆU HỌC TIẾNG ANH. ĐỌC ... layering and dyeing hair were

mountains in the Northwest, having been called the Fujiyama of America, but

it was also, and still remains, the most active volcano in the Cascade Range.

A century of volcanic inactivity has made Washingtonians complacent.

However, beneath the tranquil sylvan pandise, molten magma was slowly

rising to the suface of the earth, eventually forming a mushroom-shaped lava

dome that exploded with the force of 10 million tons of TNT at 8:30 a.m.,

throwing nature into upheaval. A hot plume of ash and debris rose 65,000 feet

into the sky, turning day into night. Billowing, hot molten rock avalanches

swept down the flanks of the mountain, mowing down everything in their

paths. Spirit Lake boiled, and rivers turned black. On the stones great swaths

of trees were blown away from the mountain and tossed in heaps. This burned

everywhere. In the aftermath, what had been pristione beauty only hours

before lay in total devastation. The crest of the mountain had been completely

blown away and a thick carpet of ash covered the landscape. Trees were

strewn about like toothpseks. There were no signs of life.

Most people believed that decades, even centuries, would pass before

the land would recover. However, nature proved to be far more resilient than

apected. The return of life, both plant and animal, was remarkable, and today

undergrowth carpets the ground and wildlife is abundant. A forest of young

trees graces the slopes and valleys below the volcano, and a delicate and

screne beauty has returned once more to this vast wilderness area.

45. According to the passage, Mt. St. Helens was called the Fujiyama of

America because of its

(A) height

(B) beauty

(C) volcanic activity

(D) cataclysmic eruption

46. According to the passage, what was the mood of Washingtonians before

the eruption occurred?

Page 24: Tài liệu học tiếng anh - saomaidata.orgsaomaidata.org/library/306.TaiLieuHocTiengAnh.docx  · Web viewTÀI LIỆU HỌC TIẾNG ANH. ĐỌC ... layering and dyeing hair were

(A) They had been expecting the eruption for some time.

(B) They didn't know that Mt. St. Helens was an active volcano.

(C) They knew that whatever happened, nature would recover.

(D) They were not concerned about the eruption at all.

47. According to the passage. how long had Mt. St. Helens been inactive?

(A) Ten years

(B) Fifty years

(C) One hundred years

(D) Two bundred years

48. When Mt. St. Helens exploded, which of the following did NOT occur?

(A) Molten rock avalanches flowed down mountainsides.

(B) A plume of ash and debris rose to the sky.

(C) The carth cracked and formed a new valley.

(D) Fires burned in the forests.

49. According to the passage, what does the Mt. St. Helens area look like

today?

(A) Nature has made a surpnsing recovery.

(B) The land is as devastated as it was the day of the eruption.

(C) Nature has not proven to be very resilient.

(D) There is undergrowth but no trees.

LOCATING DETAILS

In some detail questions you are asked where in the passage a particular

piece of information is located. The answers to this type of question are line

numbers. The following are examples of questions for locating details:

At what point in the passage does the author discuss...?

Page 25: Tài liệu học tiếng anh - saomaidata.orgsaomaidata.org/library/306.TaiLieuHocTiengAnh.docx  · Web viewTÀI LIỆU HỌC TIẾNG ANH. ĐỌC ... layering and dyeing hair were

Where in the passage does the author first mention...?

In which lines does the author explain...?

To answer such questions, scan the passage looking for the key words using

the same technique as for detail questions.

Read the sample reading passage again (p.4).

QUESTION

1. Where in the passage does the author first mention how the test is given?

(A) Lines 3-4

(B) Lines 4-5

(C) Lines 5-7

(D) Lines 7-9

ANSWER

Answer (A) is incorrect because lines 3-4 only mention another name for lie

detectors. Answer (B) is also incorrect because in lines 4-5 only the purpose

or aim of lie detectors is mentioned. Answer (C) also is incorrect because lines

5-7 state only what the machine records, not how it works. The best answer to

the question is (D), lines 7-9, which state that "you are electronically

connected to the machine and asked a few neutral questions" and are

therefore the first mention of how a polygraph works.

QUESTION

1. In which lines does the author explain how some peopk learn to trick the

polygraph?

(A) Lines 17-19

(B) Lines 19-20

(C) Lines 20-22

(D) Lines 23-26

Page 26: Tài liệu học tiếng anh - saomaidata.orgsaomaidata.org/library/306.TaiLieuHocTiengAnh.docx  · Web viewTÀI LIỆU HỌC TIẾNG ANH. ĐỌC ... layering and dyeing hair were

ANSWER

Answer (A) is incorrect because lines 17-19 state how the machine can be

unreliable. Answer (B) is also incorrect since lines 19-20 mention only

innocent people and not how they can trick the machine. Answer (C) is also

incorrect because lines 20-22 only mention how innocent people may react to

the machine. The best answer is (D), lines 23-26, which state how "some

practiced liars" learn to "beat the machine" and in this way trick the polygraph.

EXERCISE 3Read the following passages and answer the questions on locating details that

follow.

QUESTIONS 1-7

Mineral King, located at the southern edge of Sequoia National Park in

California, is a glacially carved valley situated along the headwaters of the

east fork of the Kaweah River, at an altitude of 7,800 feet. The steep, sparsely

forested slopes of rusty mineral-rich rock surrounding the valley gave Mineral

King its name and twice nearly destroyed its isolated tranquility.

The first instance occuned in 1872 after a hunter named Harry O’Farrell

spied silver and minenl deposits and hastily staked his claim. Within a year,

ninety-three prospectors had filed claims and the Mineral King Mining District

was formed. During the rest of the decade the valley resembled a boomtown,

complete with assay office, bakeshop, barber-shop, post office, genenl store,

and cabins, and the population rose to 300. A toll road, tramway, and smelter

were built, but only one silver ingot was even produced. Moreover, year after

year winter avalanches hit the 15 mines, destroying cabins, shops, the stamp

mill, and the tramway. Discounged, the miners fmauy admitted defeat in 1881.

For years the area slumbered serencly in its obscunty, until 1969, when

the Forest Service granted a permit to Disney Enterprises, which had plans for

a momumental project replete with an Alpine village of hotels, theaters and

restaurants, a ski area designed to serve 10,000 people a day, and proposals

Page 27: Tài liệu học tiếng anh - saomaidata.orgsaomaidata.org/library/306.TaiLieuHocTiengAnh.docx  · Web viewTÀI LIỆU HỌC TIẾNG ANH. ĐỌC ... layering and dyeing hair were

for a cog railway, an aerial tramway, and a monorail. Environmentalists and

wilderness enthusiasts were horrified, and ten years of court battles ensued. It

was Nature, however, who had the final say, delivering avalanche after

avalanche over cabins, snow denection barriers, and even a platform-

mounted gun that was meant to trigger slides when they were still small. In

1978 Disney abandoned its grand plan and Mineral King was added to

Sequoia National Park, its raw beauty and isolated tranquility protected

forever.

1. At what point in the passage does the author specifically discuss the

reasons why the miners left Mineral King?

(A) Lines 10-12

(B) Lines 14-16

(C) Line 17

(D) Lines 26-28

2. Where in the passage does the author first mention Disney's plans for

Minenl King?

(A) Line 17

(B) Lines 18-21

(C) Lines 22-23

(D) Lines 26-28

3. In which lines does the author first describe Mineral King?

(A) Lines 1-3

(B) Lines 3-6

(C) Lines 7-8

(D) Lines 9-10

4. Where in the passage does the author mention the deciding factor in the

final outcome of the Disney plans?

Page 28: Tài liệu học tiếng anh - saomaidata.orgsaomaidata.org/library/306.TaiLieuHocTiengAnh.docx  · Web viewTÀI LIỆU HỌC TIẾNG ANH. ĐỌC ... layering and dyeing hair were

(A) Lines 17-19

(B) Lines 22-23

(C) Lines 23-26

(D) Lines 26-28

5. At what point in the passage does the author describe the look of Mineral

King Valley during its occupation by the miners?

(A) Lines 3-6

(B) Lines 9-10

(C) Lines 10-12

(D) Lines 15-16

6. Where in the passage does the author describe the reaction of environ

mentalists to the Disney proposal?

(A) Lines 15-16

(B) Lines 22-23

(C) Lines 23-26

(D) Lines 26-28

7. In which lines does the author explain how Mineral King got its name?

(A) Lines 1-3

(B) Lines 3-6

(C) Lines 7-8

(D) Unes 7-10

QUESTIONS 8-13

Line Between 1607 and 1732 permanent English settlements were

established along the eastem coast of North America. The new colonies

provided havens for immigrants avoiding persecution and punishment,

business failures, or poor prospects for trade and work in the mother country.

Page 29: Tài liệu học tiếng anh - saomaidata.orgsaomaidata.org/library/306.TaiLieuHocTiengAnh.docx  · Web viewTÀI LIỆU HỌC TIẾNG ANH. ĐỌC ... layering and dyeing hair were

The English government authorized the use of two kinds of agencies to

promote the establishment of settlements overseas: the chartered trading

company and the proprietorship.

The commercial joint-stock companies operating under royal charters

were composed of adventuresome stockholders, who shared the profits and

losses of their colonial venture. Two of the colonies established by English

chanered trading companies were Jamestown, Virginia, and the Puntan

colony of Massachusetts. The chief characteristic that distinguished the

corporate colony from others was the large measure of seft-government it

enjoyed. Qualified voters in the colonies chose the governor, the governor's

council, and the legislative assembly.

Of the thirteen English colonies, seven were founded as proprietorships.

Maryland, New Hampshire, New Jersey, the Carolinas, Pennsylvania, and

Georgia. The propriety charters normally granted huge tracts of land to an

individual, often royaty, or a group of persons on terms similar to feudal

tenure. Pohtical control was put in the hands of those who received the royal

grant, although in most cases it was delegated in part to representatives

chosen by the colonists.

8. Where in the passage does the author defme the commercial joint-stock

companies?

(A) Lines 4-7

(B) Lines 8-10

(C) Lines 12-14

(D) Lines 14-15

9. At what point in the passage does the author give the names of the colonies

established by proprietorship

(A) Lines 2-4

(B) Lines 10-12

Page 30: Tài liệu học tiếng anh - saomaidata.orgsaomaidata.org/library/306.TaiLieuHocTiengAnh.docx  · Web viewTÀI LIỆU HỌC TIẾNG ANH. ĐỌC ... layering and dyeing hair were

(C) Lines 16-18

(D) Lines 18-20

10. In which lines does the author explain how the corporate colony was

different from other colonies?

(A) Lines 12-14

(B) Lines 14-15

(C) Lines 18-20

(D) Lines 20-22

11. Where in the passage does the author indicate when permanent English

settlements were established in North America?

(A) Lines 1-2

(B) Lines 2-4

(C) Lines 4-6

(D) Lines 10-12

12. In what lines does the author explain proprietorships?

(A) Lines 12-14

(B) Lines 16-18

(C) Lines 18-20

(D) Lines 20-22

13. Where in the passage does the author discuss the reasons why

immigrants came to America?

(A) Lines 1-2

(B) Lines 2-4

(C) Lines 4-7

(D) Lines 8-10

Page 31: Tài liệu học tiếng anh - saomaidata.orgsaomaidata.org/library/306.TaiLieuHocTiengAnh.docx  · Web viewTÀI LIỆU HỌC TIẾNG ANH. ĐỌC ... layering and dyeing hair were

Chapter 2: RAEDING FOR MAIN IDEASIntroducing Main ideas

PREREDING QUESTIONS

Answer fhe following questions.

1. Why do you think people exercise?

2. What things should you do to be healthy?

3. What foods should you eat to be healthy?

A healthful lifestyle leads to a longer, happier, healthier life. Staying

healthy means eating a well-balanced diet, getting regular exercise, and

avoiding things that are bad for the body and mind.

Nutrition plays a key role in maintaining good health and preventing

many diseases. In spite of all the information available about diets, scientists

still believe that good nutrition can be simple. There are several basic rules to

follow. Keep fat intake low. Eat foods high in carbohydrates, which are the

starches in grains, legumes (beans and peas), vegetables, and some fruits.

Avoid too much sugar. Limit salt. Eat lots of fruits and vegetables, which are

high in vitamins.

A healthful lifestyle in an active lifestyle. Lack of proper physical

exercise can cause tiredness, irritability, and poor general health. Physical

fitness requires both aerobic exercise, such as running, bicycle riding, and

swimming, and musele-strengthening exercises, such as weight lifting.

Finally, good health is acquired by saying no to bad habits such as

smoking, drinking, and overeating and by avoiding situations that are

constantly stressful. People can take their lives and happiness into their own

hands. Maintaining a healthy lifestyle is the first step.

Page 32: Tài liệu học tiếng anh - saomaidata.orgsaomaidata.org/library/306.TaiLieuHocTiengAnh.docx  · Web viewTÀI LIỆU HỌC TIẾNG ANH. ĐỌC ... layering and dyeing hair were

EXERCISE 1Answer fhe following questions.

SKIMMING

Skim or read the passage over quickly. Do not read each detail carefully.

Usually, hut not always, the main idea is found in the first sentence of a

paragraph.

Underline the main idea in paragraph 2.

Underline the main idea in paragraph 3.

Underline the main idea in paragraph 4.

1. What is the main idea of the whole passage?

SCANNING

Look over the passage again to find the answers to questions 2-12.

Complete the following sentences with details from the passage.

2. Three kinds of food that you should avoid having too much of are ___, ___,

and ___

3. Three problems that lack of physical exercise can cause are

___, ___, and ___

4. Two kinds of aerobic exercise mentioned in the passage are

___ and ___

5. Grains, legumes, vegetables, and some fruits all contain ___

6. Fruits and vegetables contain a lot of ___

In cach of the sentences below, underline the detail that is NOT

mentioned in the passage.

7. Smoking, drinking, excessive eating, dieting, and stressful situations are not

good for your health.

Page 33: Tài liệu học tiếng anh - saomaidata.orgsaomaidata.org/library/306.TaiLieuHocTiengAnh.docx  · Web viewTÀI LIỆU HỌC TIẾNG ANH. ĐỌC ... layering and dyeing hair were

8. Physical fitness for a healthy lifestyle includes regular walking, aerobic

cxereise, and musele-strengthening exercises.

REFERENCE AND VOCABULARY QUESTIONS.

9. What does the word "their" in its second appearance on line 17 refer to?

10. What does the word "nutrition" in line 4 mean?

11. What would be the best substitute for the word "limit" in line 9?

12. What does the word “constantly” in line 17 mean?

Main Idea Questions

One of the most frequently asked questions in the Reading

Comprehension section is about the main idea of the passage. There is

usually one such question for each reading passage. As its name suggests,

the main idea is the most important idea in the passage or what the passage

is about. Each passage has main and subordinate, or less important, ideas.

The main idea is more general than the supporting ideas or details in the

passage. The main idea may be the first sentence in the paragraph, but this is

not always the case. The main idea may appear in the middle or toward the

end.

When the main idea of a passage is not clear because each paragraph

has a different main point, a question identifying the main topic of the passage

will be asked. The following are examples of main idea questions:

What is the main idea of the passage?

What is the main idea expressed in the passage?

What does the passage mainly discuss?

With what topic is the passage mainly concerned?

The primary idea of the passage is...

The main topic of the passage is...

Page 34: Tài liệu học tiếng anh - saomaidata.orgsaomaidata.org/library/306.TaiLieuHocTiengAnh.docx  · Web viewTÀI LIỆU HỌC TIẾNG ANH. ĐỌC ... layering and dyeing hair were

If the main idea of the passage is not clear because each paragraph

has a different main point, then summarize or combine the main points of

each pangraph to find the main idea. The main idea should relate to the entire

passgge and not to just one part of it. Also, the main idea should not be so

genenal that it goes beyond the passage.

The four answer choices to the main idea questions will contain the

following types of answers:

(A) Too general

(B) True but only a detail

(C) Incorrect

(D) Correct

Sample Reading Passage

Although "lie detectors" are being used by governments, police

departments, and businesses that all want guaranteed ways of detecting the

truth, the results are not always accurate. Lie detectors are property called

emotion detectors, for their aim is to measure bodily changes that contradict

what a person says. The polygraph machine records changes in heart rate,

breathing, blood pressure, and the electrical activity of the skin (galvanic skin

response, or GSR). In the first part of the polygraph test, you are electronically

connected to the machine and asked a few neutral questions ("What is your

name?" "Where do you live?"). Your physical reactions serve as the standard

(baseline) for evaluating what comes next. Then you are asked a few critical

questions among the neutral ones (“When did you rob the bank?”). The

assumption is that if you are guilty, your body will reveal the truth, even if you

try to deny it. Your heart rate, respiration, and GSR will change abruptly as

you respond to the ineriminating questions.

That is the theory; but psychologists have found that lie detectors are

simply not reliable. Since most physical changes are the same across the

emotions, machines cannot tell whether you are feeling guilty, angry, nervous,

Page 35: Tài liệu học tiếng anh - saomaidata.orgsaomaidata.org/library/306.TaiLieuHocTiengAnh.docx  · Web viewTÀI LIỆU HỌC TIẾNG ANH. ĐỌC ... layering and dyeing hair were

thrilled, or revved up from an exciting day. Innocent people may be tense and

nervous about the whole procedure. They may react physiologically to a

certain word (''bank") not because they robbed it, but because they recently

bounced a check. In either case the machine will record a "lie." The reverse

mistake is also common. Some practiced liars can lie without flinehing, and

others learn to beat the machine by tensing museles or thinking about an

exciting experience during neutral questions.

QUESTION

1. What is the main idea of the passage?

(A) Physical reactions reveal guilt.

(B) How lie detectors are used and their reliability.

(C) Lie detectors distinguish different emotions.

(D) Lie detectors make innocent people nervous.

ANSWER

Answer (A) is not correct because it is too general and does not have

anything to do with lie detectors. Answer (C) is incorrect because lie detectors

record only physical changes in the body and not emotions. A lie detector

cannot determine whether you are angry or nervous. Answer (D) is incorrect

because although lie detectors make innocent people nervous, this is only a

detail and not the main point. The best answer is (B) since this combines the

main point of the first paragraph, which is about the use of the lie detector,

with the main point of the second paragraph, which is about the reliability of

the lie detector.

Exercises on Main Ideas

EXERCISE 2 Read the following passages and find the main idea of each one.

Strictly speaking, cartography is the drawing or compiling of maps. The

explorers and surveyors go out and make the measurements and gather the

Page 36: Tài liệu học tiếng anh - saomaidata.orgsaomaidata.org/library/306.TaiLieuHocTiengAnh.docx  · Web viewTÀI LIỆU HỌC TIẾNG ANH. ĐỌC ... layering and dyeing hair were

information from which the cartographers draw their maps. Sometimes the

fieldwork and the creation of the map are done by the same person. But when

the scope is broad and the sources of information many, maps are more often

a compilation of that information. They represent the accumulated work of

many people, brought together under the supervision of one person, the

compiler. The value of the map depends, of course, on the expertise of the

compiler, who must sift through available information, select the most accurate

data, and come up with a thoughtful and accurate synthesis of the geographic

knowledge of the region.

Strategies

- The main idea is not always the first sentence in the paragraph or

passage. It can also appear in the middle or toward the end of a paragraph.

- When the main idea is not clear because each paragraph has a main

point combine all the main points to get the main idea.

- Make sure the answer you select for the main idea question relates to

the whole passage and not just to one part of it. You can scan the passage to

see whether the main idea you have selected is discussed all through the

passage.

- The wrong choices for main idea questions may be one of the

following:

1. True statements that focus on one paragraph or a detail

2. Statements that are too general and go beyond the passage

3. Statements that are incorrect misinterpretations of the main idea

Or, brefly:

1. Too specific

2. Too general

3. Incorrect

Page 37: Tài liệu học tiếng anh - saomaidata.orgsaomaidata.org/library/306.TaiLieuHocTiengAnh.docx  · Web viewTÀI LIỆU HỌC TIẾNG ANH. ĐỌC ... layering and dyeing hair were

1. What is the main idea of the passage?

(A) The definition of cartography is the drawing or compiling of maps.

(B) Maps are the product of a group effort brought together usually by

one person.

(C) Not all of the infonnation initially compiled for maps is accurate.

(D) The compiler's task is more important than that of the explorers and

surveyors.

In the 1820s and 1830s American painting added a new chapter to the

story of its development. Until the nineteenth century, portraiture and

occasional historical pieces were the only concerns of American art, but

throughout the 1800s some of America's most talented painters chose to

depict landscapes and the daily activities of ordinary people. With the nation's

declaration of independence had come prosperity and with it the opportunity

and inclination for painters to contemplate their environment. As they traveled

beyond the early settlements and left the nation's first cities, such as Boston

and Philadelphia, they began to experience and appreciate the pristine beauty

of the American scenery, which differed greatly from the European landscape,

partly because in its unsettled state it appeared wild and pnmeval.

2. What is the main topic of the passage?

(A) Conditions in the early 1800s were favorable to the emergence of

the American landscape artist.

(B) In the early 1800s, landscapes were produced in preference to

portraits and historical pieces.

(C) America's declaration of independence brought prosperity to the

nation and with it an appreciation of the outdoors.

(D) An increase in travel in America led to an appreciation of the beauty

of the American landscape.

Page 38: Tài liệu học tiếng anh - saomaidata.orgsaomaidata.org/library/306.TaiLieuHocTiengAnh.docx  · Web viewTÀI LIỆU HỌC TIẾNG ANH. ĐỌC ... layering and dyeing hair were

Speculation about the carth's crust has a special edge of urgency in

California, which sits on the San Andreas fault, the world's most famous and

respected fracture zone. Not surprisingly, it was a scientist at the California

Institute of Technolog, Charles F. Richter, who invented the Richter scale

used to measure earthquakes. Seismic activity in California is being constantly

monitored and mapped. Seismometers register many thousands of small

earthquakes every year, and computers instantly calculate the location, depth,

and magnitude of an earthquake. Laser distance-ranging networks can detect

changes of length, indicating change in crustal stress, accurate to about half

an inch in 20 miles. Satellite measurements of crustal blocks are improving,

and California seismologists believe they may in time be precise enough to

allow earthquake prediction.

3. What is the main idea expressed in the passage?

(A) The Richter scale was invented in Califomia.

(B) Computers provide a variety of information about earthquakes.

(C) A great deal of attention is paid to earthquake activity in Califomia.

(D) Earthquake prediction will be possible in the future.

The first expedition down the Colorado River was made by John Wesley

Powell and his party in 1869. Powell had made long trips down the Ohio and

the Mississippi and its tributanes during his twenties, when his lifelong interest

in natural history developed. In 1867 he led his first expedition to the Rockies,

a couccting trip for the museum he had founded in Illinois. While on Pike's

Peak, near Colorado Springs, Powell conceived his great plan to explore the

Colorado River. On May 24, 1869, he and his party set off down the upper

Colorado and nothing was heard from them for thirty-seven days. During that

time Powell and party braved uncharted territory, encounters with the natives,

fierce rapids, and 20-foot waterfalls, as they followed the Colorado through the

Grand Canyon to the Gulf of California.

4. What is the main subject of the passage?

Page 39: Tài liệu học tiếng anh - saomaidata.orgsaomaidata.org/library/306.TaiLieuHocTiengAnh.docx  · Web viewTÀI LIỆU HỌC TIẾNG ANH. ĐỌC ... layering and dyeing hair were

(A) Powell was uniqucly qualifled to lead an expedition down the

Colorado.

(B) Powell was inspired to explore the Colorado whilc on Pike's Peak.

(C) People were concerned when nothing was heard from Powell and

his party for over a month.

(D) Powell and his party faced daunting challenges on the first Colo-

rado River expedition.

Innovations in transportation in the 1800s permitted space to be

traversed more rapidly and were crucial to the industnal expansion of the

North. The great spaces that separated producers from consumers made

speed essential, especially in the movement of perishable freight. The

development of the steam-powered locomotive in the 1850s and the rapid

extension of the railways in the 1840s and 1850s provided the answer to the

need for faster transport and dramatically altered patterns of economic

development throughout the United States. In 1830 there were 32 miles of

rails in the country, in 1840 there were 2,818 miles, and by 1850 there were

more than 9,000 miles. The rapid atension of rail mileage enabled the

railroads significantly to reduce their costs for shipping freight and carrying

passengers, thus enabling them to price their services more cheaply and

competitively. The extension of trunk lines, into which short or local lines fed,

further tightened the east-west flow of commerce and bound the Northeast

and the old Northwest together with bands of steel.

5. What is the main theme in the passage?

(A) Railroad made the transportation of perishable freight possible.

(B) Between 1830 and 1850 over 8,000 miles of railroad track were laid.

(C) Railroads provided an important link between the Northwest and the

Northeast.

Page 40: Tài liệu học tiếng anh - saomaidata.orgsaomaidata.org/library/306.TaiLieuHocTiengAnh.docx  · Web viewTÀI LIỆU HỌC TIẾNG ANH. ĐỌC ... layering and dyeing hair were

(D) Railroads had a profound effect on the economic development of

the United States in the nineteenth century.

The few places left on earth that have not been altered by humankind

are almost invanably hostile to humans. One such place is the Alaskan Arctic,

which is inhabited, where inhabited at all, by only a scattering of Eskimos,

Native Americans, and whites. But while the Arctic is indeed a chill and

inimical realm of snow, ice, and polar bears, it is also a region of great beauty

and, above all, purity where plants and animals still exist undisturbed in a

state of natural balance. Nearly one third of Alaska lies north of the Arctic

Circle and consists of pristine land. The Brooks Range cuts across the region

like a wall, making access difficult. Even today, in an age of let travel, the

number of persons who have had firsthand experience in the Alaskan Arctic

remains small, and countless valleys and mountains go unnamed and even

unexplored.

6. What is the primary topic of the passage?

(A) The Alaskan Arctic is a beautifully pristine realm of snow, ice, and

polar bears.

(B) The Alaskan Arcti is habitable only to Arctic animals and a few hardy

humans.

(C) The ruggedness of the Alaskan Arctic makes it one of the last few

remaining pristine areas in the world.

(D) Remarkably, parts of the Alaskan Arctic still remain unexplored.

In the first half of the nineteenth century, the first distinctly American

culture took form. The rise of an American tradition in literature paralleled the

expansion of the nation, as American writers began to look within themselves

and across their enlarged continental homeland for their subjects and themes.

The romance, or novel, provided a useful form for dealing with the large moral

subjects and the peculiar circumstances of the American setting. In James

Fenimore Cooper's The Pioneers (1823) and The Deerslayer (1841). Natty

Page 41: Tài liệu học tiếng anh - saomaidata.orgsaomaidata.org/library/306.TaiLieuHocTiengAnh.docx  · Web viewTÀI LIỆU HỌC TIẾNG ANH. ĐỌC ... layering and dyeing hair were

Bumppo and the Mohican guide Chingachgook confronted the environment of

the American frontier, chronicling the advance of "civilization" and questioning

the implications of its impact on the natural world. The theme of the individual

confronting nature was further developed by Hennan Melville in the classic

novel Moby Dick (l951). Nathaniel Hawthome dealt with equally difflcult

questions of inner limits and the individual's responsibilities to society in The

Scarlet Letter (1850) and The House of the Seven Gables (1851).

7. What is the main idea expressed in the passage?

(A) As the nation expanded, novelists began writing about the American

frontier.

(B) The first American literature took the form of novels that dcalt

with uniquely American themes.

(C) In their novels, Melville and Hawthome both addressed difficult

questions facing Americans.

(D) The individual versus nature was one of the main themes explored

in carly American literature.

Because different tree species adapted to difierent climates and soil

types have evolved over millennia, many kinds of forests occupy the earth

today. The primitive forests of several hundred million years ago consisted of

fewer kinds of trees. In fact, the earliest "trees", which grew nearly 500 million

years ago, were like giant club mosses. They lacked true roots and consisted

of a tangle of specialized branches that clambered over rocky ground. Fifty

million years later came the dense forests of tree ferns that prevailed in

tropical climates of that crab. The forerunners of modem conifers were on the

scene 300 milhon years ago, when plant life abundantly colonized marshy

land, building the tremendous coal and oil reserves so imponant today. By the

time the dinosaurs roamed the earth some 180 million years ago, during the

Cretaceous period, seed-bearing trees had evolved that shed their leaves in

Page 42: Tài liệu học tiếng anh - saomaidata.orgsaomaidata.org/library/306.TaiLieuHocTiengAnh.docx  · Web viewTÀI LIỆU HỌC TIẾNG ANH. ĐỌC ... layering and dyeing hair were

winter; from these have sprung the angiosperms and our present deciduous

forests.

8. What is the main idea of the passage?

(A) Conifers are the oldest trees in today's forests.

(B) Climate affected the development of trees over millennia.

(C) The predecessors of today's forests were giant club mosses and

tree ferns.

(D) The variety of trees in today's forest are a result of millions of years

of evolution.

Birds have two basic types of sounds: songs and calls. Songs are

usually more complex than caus and are utihzed primarily by adult males

during the breeding season to establish temtories or attract mates. Caus are

normally simple notes, single or repeated, vocalized by males and females in

all seasons to express alarm or maintain contact with mates, offspring, or

other birds of the same species. All songs and most calls are distinctive, and

with concentrated study and practice? bird-watchers can learn to identify

many birds by their sounds and to call them as well.

9. What is the main idea of the passage?

(A) Bird calls and songs are distinctive, meaningful, and identifiable.

(B) Bird songs are complicated and used mainly by males to attract

mates.

(C) Birds have their own language by which they maintain contact.

(D) Bird-watchers can identify many bird calls and their meanings and

learn to mimic them as well.

Hawaii was originally settled by the natives of the South Pacific, who

arrived in the islands in canoes laden with breadfruit, yams, taro, coconut,

bananas, pigs, and chickens. Supplementing these foods were over a

hundred different edible fishes and forty kinds of seaweed from the

Page 43: Tài liệu học tiếng anh - saomaidata.orgsaomaidata.org/library/306.TaiLieuHocTiengAnh.docx  · Web viewTÀI LIỆU HỌC TIẾNG ANH. ĐỌC ... layering and dyeing hair were

surrounding waters. Hawaiian food was eaten raw or wrapped in taro leaves,

seasoned with coconut, and cooked.

In the early 1800s, the whalers and missionaries introduced stews,

chowders, cumes, comed beef, dried beef, salt salmon, and Indian and

comstarch puddings. Most likely, pipkaula (jerked beef), lomi lomi salmon, and

haupia (coconut pudding) evolved during this period.

In the late nineteenth century immigrants from China, Japan, and Korea

were brought to Hawaii to work the sugar plantations. The Chinese brought

rice, soybeans, and vegetables and their ways of cooking them. The

Japanese brought sukiyaki and teriyaki, among many other foods. Settlers

from the continental United States also brought their favorite recipes and in

the spint of aloha, the Hawaiians have accepted each group's offerings and

drawn the best from them. Thus, a Hawaiian feast is a gastronomic

experience, the essence of Hawaii and its many cultures.

10. What is the main topic of the passage?

(A) Whalers and missionanes introduced new kinds of foods to the

people of Hawaii.

(B) Sugar plantations were worked by immigrants from Asia, who

brought their native foods with them.

(C) Hawaiian rood is a combination of the foods of many peoples and a

reflection of Hawaii's history.

(D) The natives of the South Pacific who first settled in Hawaii ate raw

food, whereas other immigrants cooked theirs.

Lesson 44: OVERVIEW QUESTIONS MAIN IDEA, MAIN TOPIC, AND MAIN PURPOSE QUESTIONSAfter almost every passage, the first question is an overview question about

the main idea, main topic, or main purpose of a passage. Main idea questions

ask you to identify the most important thought in the passage.

Page 44: Tài liệu học tiếng anh - saomaidata.orgsaomaidata.org/library/306.TaiLieuHocTiengAnh.docx  · Web viewTÀI LIỆU HỌC TIẾNG ANH. ĐỌC ... layering and dyeing hair were

Sample Questions

- What is the main idea of the passage?

- The primary idea of the passage is...

- Which of the following best summarizes the author's main idea?

When there is not a single, readily identified main idea, main topic

questions may be asked. There ask you what the passage is generally

“about”.

Sample Questions

- The main topic of the passage is…

- What does the passage mainly discuss?

- The passage is primarily concerned with…

Main purpose questions ask why an author wrote a passage. The

answer choices for these questions usually begin with infinitives.

Sample Questions

- The author's purpose in wnting is...

- What is the author's main purpose in the passage?

- The main point of this passage is...

- Why did the author write the passage?

Sample Answer Choices

- To define ...

- To relate ...

- To discuss ...

- To propose ...

- To illustrate ...

- To support the idea that ...

Page 45: Tài liệu học tiếng anh - saomaidata.orgsaomaidata.org/library/306.TaiLieuHocTiengAnh.docx  · Web viewTÀI LIỆU HỌC TIẾNG ANH. ĐỌC ... layering and dyeing hair were

- To distinguish between ___ and ___

- To compare ___ and ___

Don't answer the initial overview question about a passage until you

have answered the other questions. The process of answering the detail

questions may give you a clearer idea of the main idea, topic or purpose of the

passage.

The correct answers for main idea, main topse, and main purpose

questions correctly summarize the main points of the passage; they must be

more general than any of the supporting ideas or details, but not so general

that they include ideas outside the scope of the passages.

Distractors for this type of question have one of these charactenstics:

1. They are too specific.

2. They are too general.

3. They are incorrect according to the passage.

4. They are irrelevant (unrelated) to the main idea of the passage.

If you re not sure of the answer for one of these questions, go back and

quickly scan the passage. You can usually infer the main idea, main topic, or

main purpose of the entire passage from an understanding of the main ideas

of the paragraphs that make up the passage and the relationship between

them.

OTHER OVERVIEW QUESTIONS

A number of other questions ate asked that require an overall

understanding of the passage. These are often the last question in a set of

questions.

Tone questions ask you to detemine the author's feelings about the

topic by the language that he or she uses in writing the passage. Look for

vocabulary that indicates if the author's feelings are positive, negative, or

neutral.

Page 46: Tài liệu học tiếng anh - saomaidata.orgsaomaidata.org/library/306.TaiLieuHocTiengAnh.docx  · Web viewTÀI LIỆU HỌC TIẾNG ANH. ĐỌC ... layering and dyeing hair were

Sample Questions

- What tone does the author take in writing this passage?

- The tone of this passage could best be described as ...

Sample Answer Choices

- Positive

- Humorous

- Worried

- Favorable

- Negative

- Neutral

- Optimistic

- Critical

- Objective

- Pleased

- Angry

- Impersonal

- Respectful

- Defiant

- Impersonal

If you read the following sentences in passages, would the tone of those

passages most likely be positive or negative?

1. That was just the beginning of a remarkable series of performances

by this brilliant actress.

2. Despite some minor problems, this device has a number of admirable

features.

Page 47: Tài liệu học tiếng anh - saomaidata.orgsaomaidata.org/library/306.TaiLieuHocTiengAnh.docx  · Web viewTÀI LIỆU HỌC TIẾNG ANH. ĐỌC ... layering and dyeing hair were

3. This practice is a waste of time and money.

4. At the time his poems were first published, they were very popular,

but today most critics find them simplistic and rather uninteresting.

The italicized words in sentences 1 and 2 show a positive tone; in 3 and

4, the italicized words indicate a negative attitude. Notice that sentence 2

contains negative words (“minor problems”) but the overall meaning of the

sentence is positive. Sentence 4 contains positive language (“very popular”)

but overall, the tone is negative. (Words like despite, but, although, however,

and similar words can "reverse" the tone of the passage).

Most TOEFL reading passages have a neutral tone, but sometimes an

author may take a position for or against some point. However, answer

choices that indicate strong emotion - angry, outraged, sad, and so forth - will

seldom be correct.

Attiude questions very similar to tone questions. Again, you must

understand the author's opinion. The language that the author uses will tell

you what his or her position is.

What is the author’s anitude toward smoking on airplanes as expressed

in the sentence below?

Although some passengers may experience a slight discomfort from not

smoking on long flights, their smoking endangers the health of all the

passengers and crew.

The author opposes smoking during flights. He admits that there is

some argument in favor of smoking - some passengers may feel discomfort -

but this is not as important as the fact that smoking can be dangerous to

everyone on the flight. The use of the word although shows this.

Sample Questions

- What is the author's attitude toward...

- The author's opinion of ___ is best described as...

Page 48: Tài liệu học tiếng anh - saomaidata.orgsaomaidata.org/library/306.TaiLieuHocTiengAnh.docx  · Web viewTÀI LIỆU HỌC TIẾNG ANH. ĐỌC ... layering and dyeing hair were

- The author's attitude toward ___ could best be described as one of...

- How would the author probably feel about...

Another type of attitude question presents four statements and asks

how the author would teel about them.

- Which of the following recommendations would the author most likely

support?

- The author would be LEAST likely to agree with which of the following

statements?

- The author of the passage would most likely be in favor of which of the

following policies?

Organization questions ask about the overall structure of a passage or

about the organization of a particular paragraph.

Sample Question

- Which of the following best describes the organization of the passage?

Sample Answer Choices

- A genenl concept is defmed and examples are given.

- Several generalizations are presented, from which a conclusion is

drawn.

- The author presents the advantages and disadvantages of __

- The author presents a system of classification for ___

- Persuasive language is used to argue against ___

- The author describes ___

- The author presents a brief account of ___

- The author compares ___ and ___

Questions about previous or following paragraphs ask you to assume

that the passage is part of a longer work: what would be the topic of the

Page 49: Tài liệu học tiếng anh - saomaidata.orgsaomaidata.org/library/306.TaiLieuHocTiengAnh.docx  · Web viewTÀI LIỆU HỌC TIẾNG ANH. ĐỌC ... layering and dyeing hair were

hypothetical paragraph that precedes or follows the passage? To find the topic

of the previous paragraph, look for clues in the first line or two of the passage;

for the topic of the following passage, look in the last few lines. Sometimes

incorrect answer choices mention topics that have already been discussed in

the passage.

Sample Questions

- With what topic would the following / preceding paragraph most like

deal?

- The paragraph prior to / after the passage most probably discusses...

- It can be inferred from the passage that the previous / next paragraph

concerns...

- What most likely precedes / follows the passage?

EXERCISE 44.1Focus: Identifying correct answers and recognizing distractors in main

idea / main topic / main purpose questions.

Directions: Read the passages. Then mark each answer choice

according to the following system:

S Too specific

G Too general

X Incorrect

I Irrelevant

C Correct

The first one is done as an example.

There are two main types of cell division. Most cells are produced by a

process called mitosis. In mitosis, a cell divides and forms two identical

daughter cells, each with an identical number of chromosomes. Mosl one-

Page 50: Tài liệu học tiếng anh - saomaidata.orgsaomaidata.org/library/306.TaiLieuHocTiengAnh.docx  · Web viewTÀI LIỆU HỌC TIẾNG ANH. ĐỌC ... layering and dyeing hair were

celled creatures reproduce by this method, as do most of the cell in multi-

celled plants and animals. Sex cells, however, are formed in a special type of

cell division called meiosis. This process reduces the number of

chromosomes in a sex cell to half the number found in other kinds of cells.

Then, when sex cells unite, they produce a single cell with the onginal number

of chromosomes.

1. What is the main topic of this passage?

S (A) The method by which one-celled organisms reproduce

C (B) A comprason between mitosis and meiosis

X (C) Meiosis, the process by which identical cells are produced.

The last gold rush belongs as much to Canadian history as it does to

American. The discovery of gold along the Klondike River, which flows from

Canada's Yukon Territory into Alaska, drew some 30,000 fortune hunters to

the north. The Yukon became a territory and its capital of the time, Dawson,

would not have existed without the gold rush. The gold strike furnished

material for a dozen of Jack London's novels; in inspired Rohert Service to

write "The Shooting of Dan McGrew" and other poems, and it provided the

background for the wonderful Charlie Chaplin movie, The Gold Rusb. It also

marked the beginnings of modem Alaska.

2. This author’s main purpose in writing is to

___ (A) discuss the significance of mining in Canada and the United

States

___ (B) show the influence of the Klondike gold strike on the creative

arts

___ (C) point out the significance of the Klondike gold strike

The keystone arch was used by almost every early civilization. To build

a keystone arch, stones are cut so that the opposite sides taper toward each

other sightly. The upper and lower surfaces are carved so that when several

Page 51: Tài liệu học tiếng anh - saomaidata.orgsaomaidata.org/library/306.TaiLieuHocTiengAnh.docx  · Web viewTÀI LIỆU HỌC TIẾNG ANH. ĐỌC ... layering and dyeing hair were

stones are placed side by side, the upper and lower surfaces meet in smooth,

continuous curves. Some form of scaffolding is built under the arch and

shaped to accept the curved underside of the stones. Then the stones are

fitted in place one by one. The kystone is the top center stone, the last to be

dropped into position. Afterwards, the scaffolding is removed and the arch is

self-supporting.

3. The passage mainly concerns

___ (A) the basic principles of building keystone arches

___ (B) the uses of arches in modem architecture

___ (C) the role of scaffolding in bullding kyrstone arches

Circumstantial evidence is evidence not drawn from the direct

observation of a fact. If, for example, there is evidence that a piece of rock

embedded in a wrapped chocolate bar is the same type of rock found in the

vicinity of the candy factory, and that rock of this type is found in few other

places, then there is circumstantial evidence that the stone found its way into

the candy during manufacture and suggests that the candy maker was

negligent. Despite a popular notion to look down on the quality of

circumstantial evidence, it is of great usefulness if there is enough of it and if it

is properly interpreted. Each circumstance, taken singly, may mean little, but a

whole chain of circumstances can be as concluslve as direct evidence.

4. What is the main idea of the passage?

___ (A) A manufacturer's negligence can be shown by direct evidence

only.

___ (B) Enough circumstantial evidence is as persuasive as direct

evidence.

___ (C) Circumstantial evidence can be very useful in science.

The Nonhwest Ordinance was passed by Congress in 1787. It set up

the government structure of the region nonh of the Ohio River and west of

Page 52: Tài liệu học tiếng anh - saomaidata.orgsaomaidata.org/library/306.TaiLieuHocTiengAnh.docx  · Web viewTÀI LIỆU HỌC TIẾNG ANH. ĐỌC ... layering and dyeing hair were

Pennsylvania, then called the Northwest Territory. It set the conditions under

which parts of the territory could become states having equality with the older

states. But the ordinance was more than just a plan for government. The law

also guaranteed freedom of religion and trial by jury in the Territory. It

organized the territory into townships of 36 square miles and ordered a school

to be built for each township. It also abolished slavely in the Territory. The

terms were so attractive that thousands of pioneers poured into the Territory.

Eventually, the Territory became the states of Ohio, Indiana, lllinois, Michtgan,

and Wisconsin.

5. What is the main topic of this passage?

___ (A) The structure of government

___ (B) The provisions of an important law

___ (C) The establishment of schools in the Northwest Territory

The story of the motel business from 1920 to the start of Wodd War II in

1941 is one of uninterrupted growth. Motels spread from the west and the

midwest all the way to Maine and Florida. They clustered along

transcontinental highways such as U.S. Routes 40 and 66 and along the

north-south routes running up and down both the East and West Coasts.

There were 16,000 motels by 1930 and 24,000 by 1940. The motel industry

was one of the few industries that was not hurt by the Depression of the

1930s. Their cheap rates attracted travelers who had very little money.

6. What does the passage mainly discuss?

___ (A) How the Depression hurt U.S. motels

___ (B) The impact of transcontinental highways

___ (C) Two decades of growth for the motel industry

An old proverb states, "Beware of oak, it draws the stroke." This saying

is handy during thunder" stom season. In genenl, trees with deep roots that

tap into groundwater attract more lightning than do trees with shallow, drier

Page 53: Tài liệu học tiếng anh - saomaidata.orgsaomaidata.org/library/306.TaiLieuHocTiengAnh.docx  · Web viewTÀI LIỆU HỌC TIẾNG ANH. ĐỌC ... layering and dyeing hair were

roots. Oaks are around 50 times more likely to be struck than becches.

Spruces are nearly as sate as beeches. Pines are not as safe as these two,

but are stul much safer than oaks.

7. What is the author's main point?

___ (A) Old proverbs often contain important truths.

___ (B) Trees with shanow roots are more likely to avoid lightning than

those with deep roots.

___ (C) The deeper a tree's roots, the safer it is during a thunderstorm.

Altemative history is generally classmed ls a type of science fiction, but

It also bears some relation to historical fiction. This type of writing describes

an imaginary world that is identical to ours up to a certain point in history, but

at that point, the two worlds diverge; some important historical event takes

place in one world but not in the other, and they go in different directions.

Altemative histories might describe worlds in which the Roman Empire had

never fallen, in which the Spanish Armada had been victorious, or in which the

South had won the Civill War. Or they may suppose that some technology had

been introduced carrier in the world's history than actually happened. For

example: What if computers had been invented in Victorian times? Many

readers find these stones interesting because of the way they stimte the

imagination and get them thinking about the phenomenon of cause and effect

in history.

8. What is the main idea of this passage?

___ (A) Altemative histories describe worlds in which history has taken

another course.

___ (B) Altemative histones are a type of historical novel.

___ (C) Science fiction writers have accurately predicted certain actual

scientific developments.

Page 54: Tài liệu học tiếng anh - saomaidata.orgsaomaidata.org/library/306.TaiLieuHocTiengAnh.docx  · Web viewTÀI LIỆU HỌC TIẾNG ANH. ĐỌC ... layering and dyeing hair were

Until the late 1700s, metal could not be turned on a lathe to make it

uniformly smooth and round.

The operator could not gulde the cutting tool evenly by hand against the

turning piece. This problem was solved by David Wilkinson of Pawtucket,

Rhode rsland. In 1798 he invented a machine in which the cutter was clamped

into a moveable slide that could be advanced precisely, by hand crank,

paranel to the work. The slide rest, as it came to be called, has many uses. It

permits the manufacture of pans so unlform that they can be interchanged.

Without it. mass production would not have been possible. As it turns out, the

great English machinist Henry Maudsley developed nearly the same

mechanism a few years before, but this was unknown to Willdnson and does

not diminish his accomphshment.

9. Why did the author write this passage?

___ (A) To prove that Wilkinson's invention was based on Maudsley's.

___ (B) To demonstrate the importance of mass production to American

society.

___ (C) To show the usefuiness of Wilkinson's invention.

Almost every foml of transportation has given some one the idea for a

new type of toy. After the Montgomer brothers new the first balloon, toy

balloons became popular playthings. In the nineteenth century, soon after

railroads and steamships were developed, every child had to have model

trains and steamboats. The same held true for automobues and airplanes in

the early twentieth century. Toy rockets and missiles became popular at the

begining of the space age, and by the 1980s, there were many different

versions of space-shuttle toys.

10. The main idea of this pusage is that

__ (A) inventors have been inspired by toys to build new forms of

transponation

Page 55: Tài liệu học tiếng anh - saomaidata.orgsaomaidata.org/library/306.TaiLieuHocTiengAnh.docx  · Web viewTÀI LIỆU HỌC TIẾNG ANH. ĐỌC ... layering and dyeing hair were

__ (B) toy automobues and airplanes were verlr popular in the early

1900s

__ (C) toy design has followed developments in transponation

EXERCISE 44.2Focus: Answering a variety of overview questions about short passages.

Directions: Read the passages and mark the best answer choice -(A), (B), (C),

or (D).

American folk music originated with ordinary people at time when the

rural population was isolated and music was not yet spread by radio, records,

or music videos. It was transmitted by oral tradition and is noted for its energy,

humor, and emotional impact. The maior source of early American folk songs

was music from the British Isles, but songs from Africa as well as songs of the

American Indian have significant part in its heritage. Later settlers from other

countries also contributed songs. In the nineteenth century, composer Steven

Foster wrote some of the most enduringly popular of all American songs,

which soon became part of the folk tradition. Beginning in the 1930s, Woody

Guthrie gained great popularity by adapting tradaional melodies and lyrics and

Supplying new ones as well. In the 1950s and 1960s, signer-composers such

as Pete Secger, Bob Dylan, and Joan Baez continued this tradition by creating

“urban” folk music. Many of these songs dealt with important social issues,

such as nacial integration and the war in Vietnam.

1 . The primary purpose of this passage is to

___ (A) trace the development of American folk music

___ (B) explain the oral tradition

___ (C) contrast the styles of folk musicians

___ (D) point out the influence of social issues on "urban" folk music

Page 56: Tài liệu học tiếng anh - saomaidata.orgsaomaidata.org/library/306.TaiLieuHocTiengAnh.docx  · Web viewTÀI LIỆU HỌC TIẾNG ANH. ĐỌC ... layering and dyeing hair were

Every scientific discipline tends to develop its own special language

because it finds ordinary words inadequate, and psychology is no different.

The purpose of this special jargon is not to mystify non-psychologists; rather, it

allows psychologists to accurately describe the phenomena they are

discussing and to communicate with each other effectively. Of course,

psychological terminology conslsts in part of everyday words such as emotion,

intelligence, and motivation, but psychologists use these words some-what

differently. For example, laymen use the term anxiety to mean nervousness or

fear, but most psychologists reserve the term to describe a condition produced

when one fears events over which one has no control.

2. The main topic of this passage is

___ (A) effective communication

___ (B) the special language of psychology

___ (C) two definitions of the word "anxiety"

___ (D) the jargon of science

Gifford Pinchot was the first professlonauy trained forester in the United

States. After he graduated from Yale in 1889, he studied forestry in Europe. In

the 1890s he managed the forest on the Biltmore estate in North Carolina

(now Pisgah National forest) and became the first to pracuce scientme

forestry. Perhaps his most important contribution to conservation was

persuading President Theodore Roosevelt to set aside millions of acres in the

West as forest reserves. These lands now make up much of the national

parks and national forests of the United States. Pinchot became the chief

forester of the U.S. Forest Service in 1905. Although he held that post for only

five years, he established guidelines that set forest policy for decades to

come.

3. The passage primarily dcals with

___ (A) Gifford Pinchot's work on the Bilưnore Estate

Page 57: Tài liệu học tiếng anh - saomaidata.orgsaomaidata.org/library/306.TaiLieuHocTiengAnh.docx  · Web viewTÀI LIỆU HỌC TIẾNG ANH. ĐỌC ... layering and dyeing hair were

___ (B) the practice and theory of scientific forestry

___ (C) the origin of national parks and national forests in the United

States

___ (D) the contributions Gifford Pinchot made to American forestry

Off-Broadway theater developed in New York City in about 1950 as a

result of dissatisfaction with conditions on Broadway. Its founders believed

that Broadway was overly concerned with producing safe, commercially-

successful hit plays rather than drama with artistic quality. Off-Broadway

producers tried to assist playwrights, directors, and performers who could not

find work on Broadway. Off-Broadway theater were poorly equipped, had

limited seating, and provided few conveniences for audiences. But, the

originality of the scripts, the creativity of the performers, and the low cost of

tickets made up for these disadvantages, and off-Broadway theater

prospered. However, by the 1960s, costs began to rise, and by the 1970s, off-

Broadway theater was encountering many of the difficulties of Broadway and

had lost much of its vitality. With its decline, a experimental movement called

off-off-Broadway theater developed.

4. What is the main idea of this passenge?

___ (A) After initial success, off-Broadway theater began to decline.

___ (B) Off-Broadway theaters produced many hit commercial plays.

___ (C) Theaters on Broadway were not wen equipped.

___ (D) Off-Broadway plays were highly creative.

5. The pagraph that follows this passage most likely deals with

___ (A) the help off-Broadway producers provided directors,

playwrights, and performers

___ (B) methods off-Broadway theaters used to cope with rising prices

(C) the development of off-off-Broadway theater

Page 58: Tài liệu học tiếng anh - saomaidata.orgsaomaidata.org/library/306.TaiLieuHocTiengAnh.docx  · Web viewTÀI LIỆU HỌC TIẾNG ANH. ĐỌC ... layering and dyeing hair were

(D) the decline of Broadway theater

At the time of the first European contact, there were from 500 to 700

languages spoken by North American Indians. These were divided into some

60 language families, with no demonstrable genetic relationship among them.

Some of these families spread across several of the seven cultural areas. The

Algonquin family, for instance, contained dozens of languages and occupied a

vast territory. Speakers of Algonquin languages included the Algonquins of the

Eastern Woodland, the Blackfoots of the Plains, and the Wiyots and Yutoks of

California. Other language families, like the Zuni family of the Southwest,

occupy only a few square miles of area and contain only a single tribal

language.

6. What is the main idea of this passage?

___ (A) Each of the cultural areas was dominated by one of the

language families.

___ (B) The Zuni languages closely related to the Algonquin language.

___ (C) There is considerable diversity in the size and the number of

languages in language families of the Nonh American Indians.

___ (D) Contact with Europeans had an extraordinary effect on the

languages of the Indian tribes of North America.

Other major changes in journalism occuned around this time. In 1846,

Richard Hoe invented the steam cylinder rotary press, making it possible to

print newspapers faster and cheaper. The development of the telegraph made

possible much speedier collection and distribution of news. Also in 1846, the

first wire service was organized. A new type of newspaper appeared around

this time, one that was more attuned to the sipirit and needs of the new

America. Although newspapers continued to cover politics, they came to

report more human interest stones and to record the most recent news, which

they could not have done before the telegraph. New York papers, and those of

Page 59: Tài liệu học tiếng anh - saomaidata.orgsaomaidata.org/library/306.TaiLieuHocTiengAnh.docx  · Web viewTÀI LIỆU HỌC TIẾNG ANH. ĐỌC ... layering and dyeing hair were

other northern cities, maintained corps of correspondents to go into all parts of

the country to cover newsworthy events.

7. The main purpose of the passage is to

___ (A) present a brief history of American journalism

___ (B) outline certain developments in mid-nineteenth-century

journalism

___ (C) explain the importance of the steam cylinder rotary press

___ (D) present some biographical information about Richard Hoe

8. What is the most probable topic of the paragaph preceding this one?

___ (A) Other types of rotary presses

___ (B) Alternatives to using wire services

___ (C) Newspapers that concentrated on polities

___ (D) Other developments in joumlism

9. The tone of the passage could best be described as

(A) objective

(B) optimistic

(C) angry

(D) humorous

In the western third of North America, the convoluted folds of the Earth's

suface and its fractured geologic structure tend to absorb the seismic energy

of an earthquake. Even if an earthquake measuring 8.5 on the Richter scale

struck Los Angeles, its force would fade by the time it reached San Francisco,

some 400 miles away. But, in the eastern two thirds of the continent the same

energy travels more easily. The earthquake that struck New Madrid, Missouri,

in 1811, estimated at 8 on the Richter scale, shook Washington, D.C., about

800 miles away, and was felt as far as Boston and Toronto.

Page 60: Tài liệu học tiếng anh - saomaidata.orgsaomaidata.org/library/306.TaiLieuHocTiengAnh.docx  · Web viewTÀI LIỆU HỌC TIẾNG ANH. ĐỌC ... layering and dyeing hair were

10. Which of the following best expresses the main idea of this passage?

___ (A) If a major earthquake strikes Los Angeles, it will probably

damage San Francisco as well.

___ (B) The New Madrid earthquake of 1811 was felt in Boston and

Toronto.

___ (C) The geology of the western United States is much more

complex than that of the East.

___ (D) Earthquakes travel farther in the East than in the West.

There has never been an adult scientist who has been half as curious

as any child between the ages of four months and four years. Adults

sometimes mistake this superb curiosity about everything as a lack of ability to

concentrate. The truth is that children begin to learn at birth, and by the time

they begin formal schooling at the age of 5 or 6, they have already absorbed a

fantastic amount of information, perhaps more, fact for fact, than they will

learn for the rest of their lives. Adults can multiply by many times the

knowledge children absorb if they appreciate this curiosity whilc

simultancously encouraging the children to learn.

11. With which of the following statements would the author probably agree?

___ (A) Children lack the abuity to concentrate

___ (B) Young children have a much greater curiosity than adult

scientists do.

___ (C) The first few years of school are the most important ones for

most children.

___ (D) Adults can utilize children's intense curiosity lo help children

learn more.

12. The paragraph following this one most likely deals with

___ (A) ways in which adults can help children learn by stimulating their

curiosity

Page 61: Tài liệu học tiếng anh - saomaidata.orgsaomaidata.org/library/306.TaiLieuHocTiengAnh.docx  · Web viewTÀI LIỆU HỌC TIẾNG ANH. ĐỌC ... layering and dyeing hair were

___ (B) the learning habits of children over the age of 4

___ (C) the methods adult scientists use to study the curiosity of young

children

___ (D) ways in which adults can become as curious as children about

their environments

Settlement houses were institutions established to improve living

conditions in poor city neighbor-hoods in the late 1800s and early 1900s. They

offered health, educational, recreational, and cultural activities. The first to

open in the United States was University Settlement in New York City. It was

established by the social reformer Stanton Coit in 1886. The most famous

example was Hull House, established by the famous refomler Jane Addams in

Chicago in 1890. Settlement houses were usually staffed by idealistic young

college graduates who were eager to improve the condition of the poor.

13. The passage mainly discusses

___ (A) American citses in the late nineteenth century

___ (B) the idealism of college graduates

___ (C) settlement houses in the late 1800s and early 1900s

___ (D) the life of several American social reformers

The dancer Isadon Duncan was a daring, dynamic innovator in dance.

While she was not very successful in teaching her highly personal style of

dance to others, she taught a genention of dancers to trust their own forms of

expression. She rebelled against the rigid, formal style of classical ballet.

Inspired by the art of Greece, she usually danced barefoot in a loose, flowing

Greck tunic. She found further inspiration in nature and used dance

movements to mirror the waves of the sea and passsing clouds.

Isadora Duncan was born in San Francisco in 1878. She gave her first

performance in 1899. Early failures gave way to triumphant performances in

Budapest, Berlin, London, and finally, in 1908, back in the United Stattes. She

Page 62: Tài liệu học tiếng anh - saomaidata.orgsaomaidata.org/library/306.TaiLieuHocTiengAnh.docx  · Web viewTÀI LIỆU HỌC TIẾNG ANH. ĐỌC ... layering and dyeing hair were

lived in Europe most of her life, establishing dancing schools for children

there. She died in 1927 near Nice, France, in a freak accident, her long scarf

being caught in the wheel of an open sports car in which she was riding.

14. The author's attitude toward Isadora Duncan could best be described as

one of

___ (A) displeasure

___ (B) admiration

___ (C) compassion

___ (D) amazement

15. Which of the following best describes the organization of the passage?

___ (A) The author first discusses Isadora Duncan's style of dance and

then her life history.

___ (B) The first paragraph deals with Isadora Duncan's role as a

teacher; the second, her role as a performer

___ (C) The author first discusses Isadora Duncan’s shortcomings and

then her positive points.

___ (D) First there is an analysis of Isadora Duncan's influences and

then of her lasting contributions to dance.

Through the centures, the dream of medieval alchemists was to

discover how to turn lead and othel "base" metals into gold. Some were fakes,

but many were learned men with philosophical goals. Their quest was based

on the ancient idea that all matter consists of different proportions of just four

substances-earth, water, fire, and air. The believed that it was possible to

adjust the proportions of the elements that made up lead by chemical means

so that it turned into gold, a process called transmutation. Their experiments

were concerned with finding the substance-which they called the philosopher’s

stone-that, when added to lead, would cause this astonishing change to take

place. Alchemists also searched for the end of life, a substance that could

Page 63: Tài liệu học tiếng anh - saomaidata.orgsaomaidata.org/library/306.TaiLieuHocTiengAnh.docx  · Web viewTÀI LIỆU HỌC TIẾNG ANH. ĐỌC ... layering and dyeing hair were

cure diseases and prolong life. They failed on both counts. However, their

techniques for preparing and studying chemicals helped lay the foundation for

the modem science of chemistry.

16. Which of the following statements best summarizes the author's attiude

toward medieval alchemists

___ (A) Although they were all fakes, their made important contribution

to science.

___ (B) Their discovery of the philosopher's stone was more important

than the achievement of modern chemists.

___ (C) Although their theories were sound, they lacked the equipment

needed to accomplish their goals

___ (D) They were unable to realize their goals, but they helped prepare

the way for modern chemistry.

EXERCISE 44.3Focus: Understanding the meaning of multi-paragraph passages by identifying

the main point of each of the paragraphs.

Directions: Read the following passages and the questions about them.

Decide which of the choice best answers the question, and mark the answer.

QUESTIONS 1 -3

In most of Europe, famers' homes and outbuildings are generally

located within a villalge, and tool and animals are housed there. Every

morning, the farmers and farm laborers leave their village to work their land or

tend their animals in distant fields and return to the villge at the end of the day.

Social life thus centripetal; that is, it is focused around the community center,

the village. Only in certain parts of Quebec has this pattern been preserved in

North America.

Page 64: Tài liệu học tiếng anh - saomaidata.orgsaomaidata.org/library/306.TaiLieuHocTiengAnh.docx  · Web viewTÀI LIỆU HỌC TIẾNG ANH. ĐỌC ... layering and dyeing hair were

Throughout most of North America, different pattern was established. It

was borrowed from northern Europe, but was pushed even further in the New

World where land was cheap or even free. It a centrifugal system of social life,

with large isolated farms whose residents go to the village only to buy goods

and procure services. The independence associated with American farmers

stems from this patten of farm settlement. The American farmer is as free of

the intimacy of the village as the urbanite.

1. The main topic of the first pangraph is

___ (A) European farm products

___ (B) social life in Quebec

___ (C) the European pattem of rural settlement

2. The main topic of the second paragraph is

___ (A) the relative isolation of North American farm families

___ (B) the relationship between farmers and urbanites in North

America

___ (C) the low cost of farmland in North America

3. The main topic of the entire passage is

___ (A) a comparison of farming in northern and southern Europe

___ (B) the difference between farming in Quebec and the rest of North

America

___ (C) European influence on American agriculture

___ (D) a contrast between a centripetal system of rural life and a

centrifugal system

QUESTIONS 4-7

While fats have lately acquired a bad image, one should not forget how

essential they are. Fats provide the body's best means of storing energy, far

more efficient energy sources than either carbohy-drates or proteins. They act

Page 65: Tài liệu học tiếng anh - saomaidata.orgsaomaidata.org/library/306.TaiLieuHocTiengAnh.docx  · Web viewTÀI LIỆU HỌC TIẾNG ANH. ĐỌC ... layering and dyeing hair were

as insulation against cold, as cushioning for the intemal organs, and as

lubricants. Without fats, energy would be no way to utilize fat soluble vitamins.

Furthermore, some fats contain fatty acids that contain necessary growth

factors and help with the digestion of other foods.

An important consideration of fat intake is the ratio of saturated fats to

unsaturated fats. Saturated fats, which are derived from dairy products,

animals fats, and tropical oils, increase the amount of cholesterol in the blood.

Cholesterol may lead to coronary heart disease by building up in the erteries

of the heart. However, unsaturated fats, derived from vegetable oils, tend to

lower serum cholesterol if taken in proportion twice that of saturated fats.

The consumption of a variety of fats is necessary, but the intake of too

much fat may lead to variety of health problems. Excessive intake of fats, like

all nutritional accesses, is to be avoided.

4. The main idea of the fist paragraph is that

___ (A) fats have a bad image

___ (B) fats serve important functions in the body

___ (C) fats store food more efficently than proteins or carbohydrates

5. What is the main idea of the second paragraph

___ (A) Unsaturated fats may reduce cholesterol levels.

___ (B) The consumption of any type of fat leads to heart disease.

___ (C) Fats taken in the proper proportion may reduce serum

cholesterol.

6. The main idea of the third paragraph is that

___ (A) people are eating less and less fat today

___ (B) fats should be gradually eliminated from the diet

___ (C) excessive consumption of fats may be dangerous to one's

health

Page 66: Tài liệu học tiếng anh - saomaidata.orgsaomaidata.org/library/306.TaiLieuHocTiengAnh.docx  · Web viewTÀI LIỆU HỌC TIẾNG ANH. ĐỌC ... layering and dyeing hair were

7. With which of the following is the whole passage primarily concerned?

___ (A) The role of fats in human health

___ (B) The dangers of cholesterol

___ (C) The benefits of fats in the diet

___ (D) The imponance of good nutrition

QUESTIONS 8- 1 0

The term weathering refers to all the ways in which rock can be broken

down. It takes place because minerals formed in a particular way (say at high

temperatures, in the case of igneous rocks) are often unstable when exposed

to various conditions. Weathering involves the interaction of the lithosphere

(the Earth's crust) with the atmosphere and hydrosphere (air and water). It

occurs at different rates and in different ways, depending on the climactic and

environmental conditions. But, all kinds of weathering ultimately produce

broken minerals and rock fragments and other products of the decomposition

of stone.

Soil is the most obvious and, from the human point of view, the most

important result of the weathering process. Soil is the weathered part of the

Earth's crust that is capable of sustaining plant life. The character of soil

depends on the nature of rock from which it is formed. It also depends on the

climate and on the relative "age" of the soil. Immature soils are little more than

broken rock fragments. Over time, immature soil develops into mature soil,

which contains quantities of humus, formed from decayed plant matter.

Mature soil is darker, richer in microscopic life, and more conducive to plant

growth.

8. The first paragraph primadly describes

___ (A) the process by which rocks are broken down

___ (B) the weathering of igneous rocks

___ (C) gradual changes in the Earth's weather patterns

Page 67: Tài liệu học tiếng anh - saomaidata.orgsaomaidata.org/library/306.TaiLieuHocTiengAnh.docx  · Web viewTÀI LIỆU HỌC TIẾNG ANH. ĐỌC ... layering and dyeing hair were

9. The main topic of the second paragraph is

___ (A) a description of immature soil

___ (B) the growth of plants

___ (C) the evolution of soil

10. The main topic of the entire passage is that

___ (A) weathering breaks down rocks and leads to the development of

soil

___ (B) soils may be classifted as mature or immature

___ (C) the process of soil development is more imponant to humans

than that of weathering

___ (D) the Earth's crust is constantly changing

QUESTIONS 11-16

The fist Dutch outpost in New Nethedands was made at Fort Orange

(now Albany) in 1624; it became a depot of the fur trade. But, the most

important settlement was at the southern tip of Manhatttan, commanding the

great harbor at the mouth of the Hudson River. Peter Minuit, first governor-

general of New Netherlands, "purchased" title to the island from the Canarsic

Indians for the equivalent of $24 worth of trinkets. However, the Canarsie

Indians might be described as tourists from Brooklyn; Minuit had to make a

later payment to the group that was actually resident there.

In 1626, engineers from Holland arrived in Manhattan to construct Fort

Amsterdam. Within its rectangular walls, permanent houses were built,

replacing the thatched dwellings of the original Manhat-tanites. The fort

became the nucleus of the town of New Amsterdam. Soon Manhattan had its

first skyline: the soud outline of the fort; the flagstaff, the silhouette of a giant

windmill; and the masts of trading ships.

The Dutch West India company established dairy farms in the vicinity of

New Amsterdam. Each morning, the cattle were driven to the "Bouwerie" (now

Page 68: Tài liệu học tiếng anh - saomaidata.orgsaomaidata.org/library/306.TaiLieuHocTiengAnh.docx  · Web viewTÀI LIỆU HỌC TIẾNG ANH. ĐỌC ... layering and dyeing hair were

the Bowery), a large open common in the city. Just southwest of the Bouwerie

was the Bowang Green, a level are where the burghers played ninepins, the

ancestor of modem bowling. The Bowling Green became the site of a cattle

fair where livestock were marketed; beer and sausage were available from

booths: cheese, lace, and linen were sold by farmers' wives; and indian

women sold baskets and other handicrafts. These colorful gatherings and

other aspects of everyday life in New Amsterdam are described in Washington

Irving's rollicking book. Diedricb Knicker-bocker’s "History of New York".

The last and most powerful governor-general of New Netherlands was

Peter Stuyvesant, famous for his temper and his wooden leg. He annexed the

Swedish colony of Delaware and ordered the streets of New Amsterdam laid

out in an orderly manner and numbered. He did his best to obtain military and

financial aid from Holland against the British. When the British sent emissaries

demanding the surrender of the colony, he wanted to fight.

Four British warships, commanded by Colonel Richard Nicolls. sailed

into the harbor in 1664. The fort was long out of repair, and there was a

shortage of ammunition. Stuyvesant had no choice but to surrender. New

Netherlands became the British colony of New York, and New Amsterdam

became New York City.

11. What is the main topic of the first paragraph

___ (A) The first Dutch settlement in New Netherlands.

___ (B) Peter Minuit's acquisition of Manhattan

___ (C) Tourism in Manhattan

12. The second paragraph deals primanly wish

___ (A) The establishment of Fort Amsterdam

___ (B) The skyline of Manhattan

___ (C) the thatched houses of the Indians

13. The third paragraph mainly describes

Page 69: Tài liệu học tiếng anh - saomaidata.orgsaomaidata.org/library/306.TaiLieuHocTiengAnh.docx  · Web viewTÀI LIỆU HỌC TIẾNG ANH. ĐỌC ... layering and dyeing hair were

___ (A) aspects of everyday life in New Amsterdam

___ (B) the origin of the game of modern bowling

___ (C) Washington Irving's book about New Amsterdam

14. What does the fourth paragraph mainly discuss?

___ (A) The annexation of the Swedish colony of Delaware

___ (B) The ordering of the streects in New Amsterdam

___ (C) A description of Peter Stuyvesant and his accomplishments

15. What is the primary topic of the entire passage?

___ (A) A history of the British colony of New York

___ (B) The origin and importance of the cattle fair

___ (C) European colonization in the New World

___ (D) Fotty years of Dutch rule in New Amsterdam

Lesson 45: FACTUAL QUESTIONS, NEGATIVE QUESTIONS, AND SCANNING QUESTIONS FACTUAL QUESTIONS

Factual questions ask about explicit facts and details given in the

passage. They often contain one of the wh-question words: who, what, when,

where, why, how much, and so on.

Factual questions often begin with the phrases "According to the

passage..." or "According to the author. . . ." When you see these phrases,

you know that the information needed to answer the question is directly stated

somewhere in the passage (unlike answers for inference questions).

To answer factual questions, you have to locate and identify the

information that the question asks about. If you are not sure from your first

reading where to look for specific answers, use the following scanning

techniques.

- Focus on one or two key words as you read the stem of each

question. Lock these words in your mind.

Page 70: Tài liệu học tiếng anh - saomaidata.orgsaomaidata.org/library/306.TaiLieuHocTiengAnh.docx  · Web viewTÀI LIỆU HỌC TIẾNG ANH. ĐỌC ... layering and dyeing hair were

- Scan the passage looking for the key words or their synonyms. Look

only for these words. Do NOT try to read every word of the passage.

- It may help to use the eraser end of your pencil as a pointer to focus

your attention. Don't reread the passage completely - just look for these

words.

- When you find the key words in the passage, carefully read the

sentence in which they occur. You may have to read the sentence preceding

or following that sentence as well.

- Compare the information you read with the four answer choices.

The order of detail questions about a passage almost always follows the

order in which ideas are presented in the passage. In other words, the

information you need to answer the first detail question will usually come near

the beginning of the passage; the information for the second will follow that,

and so on. Knowing this should help you locate the information you need.

Correct answers for detail questions are seldom the same, word for

word, as information in the passage; they often contain synonyms and use

different grammatical structures.

There are generally more factual questions-twelve to eighteen per

reading section-than any other type except (on some tests) vocabulary-in-

context questions.

NEGATIVE QUESTIONS

These questions ask you to determine which of the four choices is not

given in the passage. These questions contain the words NOT, EXCEPT, or

LEAST (which are always capitalized).

- According to the passage, all of the following are true EXCEPT

- Which of the following is NOT mentioned in the passage?

- Which of the following is the LEAST likely . . .

Page 71: Tài liệu học tiếng anh - saomaidata.orgsaomaidata.org/library/306.TaiLieuHocTiengAnh.docx  · Web viewTÀI LIỆU HỌC TIẾNG ANH. ĐỌC ... layering and dyeing hair were

Scan the passage to find the answers that ARE correct or mentioned in

the passage. Sometimes the three distractors are clustered in one or two

sentences; Sometimes they are scattered throughout the passage. The

correct answer, of course, is the one that does not appear.

Negative questions often take more time than other questions.

Therefore, you may want to guess and come back to these questions if you

have time.

There are generally from three to six negative questions per reading

section.

SCANNING QUESTIONS

These questions ask you to find where in the passage some particular

information or transition is located. They are easy to identify: the answers are

usually line numbers. They are usually easy to answer too. Scanning

questions are often the last question in a set of questions about a passage.

Use the same techniques for scanning given in Part A about detail questions.

Sample Questions

- In what line does the author shift his focus to___?

- Where in the passage does the author first discuss___?

- A description of ___ can be found in…

- Where in the passage does the author specifically stress___?

- In what paragraph does the author first mention the concept of___?

In each reading section, there are generally from one to three scanning

questions.

EXERCISE 45.1Focus: Scanning passages to locate answers for factual and scanning

questions.

Page 72: Tài liệu học tiếng anh - saomaidata.orgsaomaidata.org/library/306.TaiLieuHocTiengAnh.docx  · Web viewTÀI LIỆU HỌC TIẾNG ANH. ĐỌC ... layering and dyeing hair were

Directions: for each question, locate that part of the passage in which the

answer will probably be found, and write down the line numbers in the blank at

the end of the passage. Don’t worry about answering the question itself, only

about finding the information. The first one is done as an example. Do these

scanning exercises as fast as you can.

QUESTIONS 1-7

Antlers grow from permanent knoblike bones on a deer's skull. Deer use

their antlers chiefly to fight for mates or for leadership of a herd. Among most

species of deer, only the males have antlers, but both male and female

reindeer and caribou have antlers. Musk deer and Chinese water deer do not

have antlers at all.

Deer that live in mild or cold climates lose their antlers each winter. New

ones begin to grow the next spring. Deer that live in tropical climates may lose

their antlers and grow new ones at other times of year.

New antlers are soft and tender. Thin skin grows over the antlers as

they develop. Short, fine hair on the skin makes it look like velvet. Full-grown

antlers are hard and strong. The velvety skin dries up and the deer rubs the

skin off by scraping its antlers against trees. The antlers fall off several months

later.

The size and shape of a deer's antlers depend on the animal's age and

health. The first set grows when the deer is from 1 to 2 years old. On most

deer. the first antlers are short and straight. As deer get older, their antlers

grow larger and form intricate branches.

1. How do deer primarily use their antlers? ___1-2

2. In what way are reindeer and caribou different from other types of deer?___

3. When do deer that live in temperate climates begin to grow their antlers?

___

Page 73: Tài liệu học tiếng anh - saomaidata.orgsaomaidata.org/library/306.TaiLieuHocTiengAnh.docx  · Web viewTÀI LIỆU HỌC TIẾNG ANH. ĐỌC ... layering and dyeing hair were

4. According to the article, which of the following does the skin on deer's

antlers most closely re-semble?___

5. Which of the following factors influences the size and shape of a deer's

antlers?___

6. At what age do deer get their first antlers?___

7. What happens to deer's antlers as the deer grow older? ___

QUESTIONS 8- 13

The trumpet player Louis Armstrong, or Satchmo as he was usually

called, was among the first jazz musicians to achieve intenational fame. He is

known for the beautiful, clear tone of his trumpet - playing and for his gruff,

gravelly singing voice. He was one of the first musicians to sing in the scat

style, using rhythmic nonsense syllables instead of lyrics.

Armstrong was born into a poor family in New Orleans. He first learned

to play the cornet at the age of 13, taking lessons while living in a children's

home. As a teenager, he played in a number local jazz bands in New Orleans’

rollicking nightlife district, Storyville.

In 1922, Armstrong moved to Chicago to play in Joe "King" Oliver's

band. Two years later, he joined Fletcher Henderson's band. Then, from 1925

to 1928, Armstrong made a series of records wi group called the Hot Five, the

Hot Seven, and the Savoy Ballroom Five. These records rank among the

greatest recordings in the history of jazz. They include "Cornet Chop Sucy,"

"Potato Head Blues,” and “West End Blues."

Armstrong led a big band during the 1930s and 1940s, but in 1947,

returned to playing with small jazz groups. He performed all over the world

and made a number of hit records, such as "Hello, Dolly" and "Mack the

Knife." Armstrong also appeared in a number of movies, first in New Orleans

in 1947, High Society in 1956, and Hello, Dolly in 1969.

8. What was Armstrong's nickname? ___

Page 74: Tài liệu học tiếng anh - saomaidata.orgsaomaidata.org/library/306.TaiLieuHocTiengAnh.docx  · Web viewTÀI LIỆU HỌC TIẾNG ANH. ĐỌC ... layering and dyeing hair were

9. Which of the following phrases best describes Armstrong's singing voice?

___

10. Where did Armstrong first learn to play the cornet?___

11. In what city was Joe "King" Oliver's band based? ___

12. During what period did Armstrong record some of jazz's greatest records?

___

13. What was the first movie Armstrong appeared in? ___

QUESTIONS 14-23

In 1862, during the Civil War, President Lincoln signed the Morrill Act.

The measure was named for its sponsor, Congressman (later Senator) Justin

S. Morrill of Vermont. Popularly called the Land Grai Act, it provided each

state with 30,000 acres of public land for each senator and each

representative, it had in Congress. It required that the land be sold, the

proceeds invested, and the income used to create and maintain colleges to

teach agriculture and engineering.

Although not all states used the money as planned in the act, some

thirty states did establish new institutions. Purdue University, the University of

Illinois, Texas A & M, Michigan State, and the University of California all trace

their roots to the Morrill Act. Eighteen states gave the money to existing state

universities to finance new agricultural and engineering departments. A few

gave their money to private colleges. For example, Massachusetts used much

of its funds to endow the Massachusetts Institute of Technology. One state

changed its mind. Yale University was chosen to be funded in Connecticut,

but farmers protested, and the legistature moved the assets to the University

of Connecticut.

Most students chose to study engineering. Agriculture was not even

considered a science until it had been dignified by the work of research

stations. These were established at land-grant institu-tions in 1887 by the

Hatch Act. Gradually, universities broke away from the narrow functions

Page 75: Tài liệu học tiếng anh - saomaidata.orgsaomaidata.org/library/306.TaiLieuHocTiengAnh.docx  · Web viewTÀI LIỆU HỌC TIẾNG ANH. ĐỌC ... layering and dyeing hair were

Congress had assigned them and presented a full range of academic offerings

from anthropology to zoology.

Today there are some sixty-nine land-grant institutions in all fifty states,

the District of Columb and Puerto Rico. About one in five college students in

the United States attends land-grant schools.

14. When was the Morrill Act signed?___

15. Who sponsored the Morrill Act?___

16. What position did the sponsor of the Morrill Act have at the time it was

passed?___

17. How much land did each state receive under the Morrill Act?___

18. How many states used the money in the way it was intended by

Congress?___

19. Which of these states used its money to fund a private university?___

20. Who objected to the way the Connecticut legislature initially decided to

spend its funds?___

21. What was one effect of the Hatch Act of 1887?___

22. How many land-grant institutions are in operation at present?___

23. What percent of college students in the United States currently attend

land-grant institution?___

EXERCISE 45.2Focus: Answering factual, negative, and scanning questions about

reading pusages.

Directions: Read the following passages and the questions ahout them.

Decidc which of the choices-(A), (B), (C), or (D)-best answers the question,

and mark the answer.

QUESTIONS 1-9

Page 76: Tài liệu học tiếng anh - saomaidata.orgsaomaidata.org/library/306.TaiLieuHocTiengAnh.docx  · Web viewTÀI LIỆU HỌC TIẾNG ANH. ĐỌC ... layering and dyeing hair were

Mesa Verde is the center of the prehistoric Anasazi culture. It is located

in the high plateau lands near Four Comers. Where Colorado, Utah, New

Mexico, and Arizona come together. This high ground is malestic but not

forbidding. The climate is dry but tiny streams trade at the bottom of deeply

cut canyons, where sepal and spdngs provided water for the Anasazi to

irrgate their crops. Rich red soll provided ferrtile ground for thetr crops of com,

beans, squash, tobacco, and cotton. The Anasazi domesticated the wild

turkey and hunted deer, rabbits, and mountain sheep.

For a thousand years the Anaszi lived around Mesa Verde. Although

the Anasazi are not related to the Navajos, no one knows what these Indians

called themselves, and so they are commonly refened to by their Navajo

name, Anasazi, which means “ancient ones” in the Navajo language.

Around 550 A.D., eady Anasazi-then a nomadic people eachaeologists

call the Basketmakers-began constructing permanent homes on mesa tops. In

the next 300 years, the Anasazi made rapid technological advancements,

induding the rermement of not only basket making but allso pottery making

and weaving. This phase of development is referred to as the Early Pueblo

Culture.

By the Great Pueblo Period (1100-1300 A.D.), the Anasazi population

swelled to more than 5,0OO and the architecturally ambitious cliff dwellings

came into being. The Anasazi moved from the mesa tops onto ledges on the

steep canyon walls, creating two- and three story dweninss. They used

sandstone blocks and mud mortar. There were no doors on the first floor and

people used laddera to reach the first roof. All the villages had underground

chambers called kivas. Men held tribal councus there and also used them for

secret religious ceremonies and clan meetings. Winding paths, ladders and

steps cut into the stone led from the valleys below to the ledges on which the

vilages stood. The largest settlement contained 217 rooms. One might

surmise that these dwellings were built for protection; but the Anaszi had no

known enemies and there is no sign of conflict.

Page 77: Tài liệu học tiếng anh - saomaidata.orgsaomaidata.org/library/306.TaiLieuHocTiengAnh.docx  · Web viewTÀI LIỆU HỌC TIẾNG ANH. ĐỌC ... layering and dyeing hair were

But a bigger mystery is why the Anasazi occupied these structures such

a short time. By 1300, Mesa Verde was deserted. It is conjectured that the

Anasazi abandoned their settlements because of drought, overpopulation,

crop failure, or some combination of these. They probably moved south-ward

and were incorponted into the pueblo vilages that the Spanish explorers

encountered 200 years later. Their descendants still live in the Southwest.

1. The passage does NOT mention that the Anasazi hunted

(A) sheep

(B) turkeys

(C) deer

(D) rabbits

2. The name that the Anasazi used for them- selves

(A) means "Basketmakers" in the Navajo language

(B) is unknown today

(C) was given to them by archae-ologists

(D) means "ancient ones" in the Anasazi language

3. How long did the Early Pueblo Culture last?

(A) 200 years

(B) 300 years

(C) 550 years

(D) 1,000 years

4. Where did the Anasazi move during the Great Pueblo Period?

(A) to settlements on ledges of canyon walls

(B) to pueblos in the South

(C) onto the tops of the mesas

(D) onto the floors of the canyons

Page 78: Tài liệu học tiếng anh - saomaidata.orgsaomaidata.org/library/306.TaiLieuHocTiengAnh.docx  · Web viewTÀI LIỆU HỌC TIẾNG ANH. ĐỌC ... layering and dyeing hair were

5. According to the passage, the Anasazi buildings were made primarily of

(A) mud

(B) blocks of wood

(C) sandstone

(D) the skins of animals

6. According to the passage, the Anasazi entered their buildings on the ledges

(A) by means of ladders

(B) from underground chambers

(C) by means of stone stairways

(D) through doors on the first floor

7. According to the passage, kivas were used for all the following purposes

EXCEPT

(A) clan meetings

(B) food preparation

(C) religious ceremonies

(D) tribal councils

8. According to the passage, the LEAST likely reason that the Anasazi

abandoned Mesa Verde was

(A) drought

(B) overpopulation

(C) war

(D) crop failure

9. Where in the passage does the author mention specific acomplishments of

the Basketmakers?

(A) Lines 5-6

Page 79: Tài liệu học tiếng anh - saomaidata.orgsaomaidata.org/library/306.TaiLieuHocTiengAnh.docx  · Web viewTÀI LIỆU HỌC TIẾNG ANH. ĐỌC ... layering and dyeing hair were

(B) Lines 11-13

(C) Lines 15-16

(D) Lines 18-20

QUESTIONS 10- 15

Dulcimers are musical instruments that basically consist of wooden boxes with

strings stretched over them. In one form or another, they have been around

since ancient times, probably originating with the Persian santir. Today there

are two varieties: the hammered dulcimer and the Appalachian, or mountain

dulcimer. The former is shaped like a trapezoid, has two or more strings, and

is played with wooden mallets. It is the same instrument played in a number of

old World countries. The Appalachian dulcimer is classified by musicologists

as a box zither. It is a descendant of the Pennsyl-vania Dutch scheitholt and

the French epinette. Appalachian dulcimers are paininstakingly fashioned by

artisans in the mountains of West Virginia, Kentucky, Tennessee, and Virginia.

These instrument have three or four strings and are plucked with quills or the

fingers. They are shaped like teardrops or hourglasses. Heart-shaped holes in

the sounding board are traditional. Most performers play the instruments while

seated with the instruments in their laps, but others wear them around their

necks like guitars or place them on tables in front of them. Originally used to

play dance music, Appala-chian dulcimers were popularized by performers

such as John Jacob Niles and Jean Ritchle during the folk music revival of the

1960s.

10. According to the passage, a hammered dulcimer is made in the shape of

____ (A) an hourglass

____ (B) a heart

____ (C) a trapezoid

____ (D) a teardrop

Page 80: Tài liệu học tiếng anh - saomaidata.orgsaomaidata.org/library/306.TaiLieuHocTiengAnh.docx  · Web viewTÀI LIỆU HỌC TIẾNG ANH. ĐỌC ... layering and dyeing hair were

11. According to the passage, which of the following is NOT an ancestor of the

Appala- chian dulcimer?

____ (A) the box zither

____ (B) the santir

____ (C) the scheitholt

____ (D) the epinette

12. According to the passage, how many strings does the Appalachian

dulcimer have?

____ (A) one or two

____ (B) three or four

____ (C) four or five

____ (D) six or more

13. According to the author, most performers play the Appalachian dulcimer

____ (A) while sitting down

____ (B) with the instrument strapped around their neek

____ (C) while standing at a table

____ (D) with wooden hammers

14. According to the author, what are John Jacob Niles and Jean Ritchie

known for?

____ (A) playing dance music on Appalachian dulcimers

____ (B) are artisans who design Appalachian dulcimers

____ (C) helped bring Appalachian dulcimers to the public's attention

____ (D) began the folk music revival of the 1960s

15. Where in the passage does the author describe the hammered dulcimer?

____ (A) Lines 1-2

Page 81: Tài liệu học tiếng anh - saomaidata.orgsaomaidata.org/library/306.TaiLieuHocTiengAnh.docx  · Web viewTÀI LIỆU HỌC TIẾNG ANH. ĐỌC ... layering and dyeing hair were

____ (B) Lines 3-4

____ (C) Lines 4-5

____ (D) Lines 8-10

QUESTIONS 16-20

Humanitarian Dorothea Dix was born in Hampden, Maine, in 1802. At the age

of 19, she established a school for girls, the Dix Mansion School, in Boston,

but had to close it in 1835 due to her poor health. She wrote and published the

first of many books for children in 1824. In 1841, Dix accepted an invitation to

teach classes at a prison in East Cambridge, Massachusetts. She was deeply

disturbed by the sight of mentally-ill persons thrown in the jail and treated like

criminals. For the next eighteen months, she toured Massachusetts

institutions where other mental patients were confined and reported the

shocking conditions she found to the state legislature. When improvements

followed in Massachusetts, she turned her attention to the neighboring states

and then to the West and South.

Dix’s work was interrupted by the Civil War; she served as

superintendent of women hospital nurses for the federal government.

Dix saw special hospital for the mentally in built in some fifteen states.

Although her plan to obtain public land for her cause failed, she aroused

concern for the problem of mental illness all over the United States as well as

in Canada and Europe.

Dix's success was due to her independent and thorough research, her

gentle but persistent manner, and her ability to secure the help of powerful

and wealthy supporters.

16. In what year was the Dix Mansion School clised?

____ (A) 1821

____ (B) 1824

____ (C) 1835

Page 82: Tài liệu học tiếng anh - saomaidata.orgsaomaidata.org/library/306.TaiLieuHocTiengAnh.docx  · Web viewTÀI LIỆU HỌC TIẾNG ANH. ĐỌC ... layering and dyeing hair were

____ (D) 1841

17. Why did Dorothea Dix first go to a prison?

____ (A) She taught classes there.

____ (B) She was sent there by the state legislature.

____ (C) She was convicted of a crime.

____ (D) She was doing rescarch for a book.

18. wbere was Dorothea Dix first able to bring about reforms in the treatment

of the mentally ill?

____ (A) Canada

____ (B) Massachusetts

____ (C) The West and South

____ (D) Europe

19. Dorothea Dix was NOT successful in her attempt to

____ (A) become superintendent of nurses

____ (B) pubush books for chudren

____ (C) arouse concern for the mentally ill

____ (D) obtain public lands

20. At what point of the passage does the author discuss specific reasons for

Dix's success?

____ (A) Lines 7-8

____ (B) Lines 9-10

____ (C) Lines 11-13

____ (D) Lines 14-15

QUESTIONS 21 -26

Page 83: Tài liệu học tiếng anh - saomaidata.orgsaomaidata.org/library/306.TaiLieuHocTiengAnh.docx  · Web viewTÀI LIỆU HỌC TIẾNG ANH. ĐỌC ... layering and dyeing hair were

A quilt is a bed cover made of squares of material piceed together. Each

square consists of two layers filled with a layer of wool or cotton cloth,

feathers, or down. Often, the squares are decorated with fancy stitches and

designs. According to legend, the earliest pieced quilt was stitched in 1704 by

Sarah Sedgewick Everett, wife of the governor of the Massachusetts colony.

By 1774 George Washing-ton was buying quilts in Belvoir, Virginia, to take

back to Martha in Mount Vernon. As the frontier moved westward, quilting

went along. In addition to sleeping under them, homesteaders kept out drafts

by hanging quilts over doors and windows. And if the money ran out, quilts

were used to pay debts.

For isolated pioneer women, quilts were a source of comfort. Mary

Vilman, whose family moved to Texas from Missoun in 189O, recalled the first

time she and her mother had to spend a week alone and a dust storm came

up. "The wind blew for three days and the dust was so thick that you couldn’t

see the barn. My mother quilted all day, and she laught me how to quilt. If it

hadn't been for quilting, I think we would have gone crazy."

Quilting provided an important social function for the women of the

frontier as well. At quilting bees, women met to work on quilts, and to share

the latest news.

Today, however, the homely quilt has become a coslly cultural

phenomenon. The International Quilt Festival in Houston, Texas, "world's fair

of quiltillg," attracted only 2,500 people and displayed only 20O quilts when it

began a dozen years ago. This year where were more than 20,000 visitors

and 5,000 quilts some of which sold for as much as $50,000.

21. According to legend, who made the first American quilt?

____ (A) Sarah Sedgewick Everett

____ (B) the governor of the colony of Massachusetts

____ (C) Martha Washington

____ (D) Mary Wilman

Page 84: Tài liệu học tiếng anh - saomaidata.orgsaomaidata.org/library/306.TaiLieuHocTiengAnh.docx  · Web viewTÀI LIỆU HỌC TIẾNG ANH. ĐỌC ... layering and dyeing hair were

22. Which of the following is NOT mentioned in the passage as one of the

benefits of quilts for pioneers?

____ (A) They could be used to pay debts.

____ (B) They could be used to help

____ (C) They could provide logical comfort.

____ (D) They could be worn as warm clothing

23. According to the passage, what is a “quilting bee”?

____ (A) a type of insect

____ (B) a gathering where women socialize and make quilts

____ (C) a type of quilt

____ (D) a place where people buy and sell quilts

24. Where is the International Quilt Festival held?

____ (A) Massachusetts

____ (B) Houston, Texas

____ (C) Belvoir, Virginia

____ (D) Missouri

25. How many quilts were displayed at the first International Quilt Festival?

____ (A) 200

____ (B) 2,500

____ (C) 5,000

____ (D) 20,000

26. Where in the passage does the author first begin to discuss the way in

which the public’s perception of quilts has change in modern times?

____ (A) Lines 5-6

____ (B) Line 9

Page 85: Tài liệu học tiếng anh - saomaidata.orgsaomaidata.org/library/306.TaiLieuHocTiengAnh.docx  · Web viewTÀI LIỆU HỌC TIẾNG ANH. ĐỌC ... layering and dyeing hair were

____ (C) Lines 16

____ (D) Lines 18-19

QUESTIONS 27-32

Ambient divers are, unlike divers who go underwater in submersible vehicles

or pressure resistant suits, exposed to the pressure and temperature of the

surrounding (ambient) water. Of all types of diving, the oldest and simplest is

free diving. Free divers may use no equipment at all, but most use a face

mask, foot fins, and a snorkel. Under the surface, free divers must hold their

breath. Most free divers can only descend 30 to 40 feet, but some skilled

divers can go as deep as 100 feet.

Scuba diving provides greater range than free diving. The word scuba

stands for self-contained underwater breathing apparatus. Scuba divers wear

metal tanks with compressed air or other breathing gases. When using open-

circuit equipment, a scuba diver simply breathes air from the tank through a

hose and releases the exhaled air into the water. A closed-circult breathing

device, also called a rebreather, fitters out carbon dioxide and other harmful

gases and automatically adds oxygen. This enables the diver to breathe the

same air over and over.

In surface-supplied diving, divers wear helmets and waterproof canvas

suits. Today, sophisticated plastic helmets have replaced the heavy copper

helmets used in the past. These divers get their air from a hose connected to

compressors on a boat. Surface-supplied divers can go deeper than any other

type of ambient diver.

27. Ambient divers are ones who

____ (A) can descend to extreme depths

____ (B) use submersible vehicles

____ (C) use no equipment

____ (D) are exposed to the surrounding water

Page 86: Tài liệu học tiếng anh - saomaidata.orgsaomaidata.org/library/306.TaiLieuHocTiengAnh.docx  · Web viewTÀI LIỆU HỌC TIẾNG ANH. ĐỌC ... layering and dyeing hair were

28. According to the passage, a free diver may use any of the following

EXCEPT

____ (A) a rebreather

____ (B) a snorkel

____ (C) foot fins

____ (D) a mask

29. According to the passage, the maximum depth for free divers is around

____ (A) 40 feet

____ (B) 100 feet

____ (C) 200 feet

____ (D) 1,000 feet

30. When using closed-circuit devices, divers

____ (A) exhale air into the water

____ (B) hold their breath

____ (C) breathe the same air over and over

____ (D) receive air from the surface

31. According to the passage, surface-supplied divers today use helmets

made from

____ (A) glass

____ (B) copper

____ (C) plastic

____ (D) canvas

32. Where in the passage does the author mention which type of diver can

make the deepest descents?

____ (A) Lines 2-5

Page 87: Tài liệu học tiếng anh - saomaidata.orgsaomaidata.org/library/306.TaiLieuHocTiengAnh.docx  · Web viewTÀI LIỆU HỌC TIẾNG ANH. ĐỌC ... layering and dyeing hair were

____ (B) Lines 4-5

____ (C) Lines 11

____ (D) Lines 14-15